Download as pdf
Download as pdf
You are on page 1of 431
Solutions to LE. Irodov’s Problems in General Physics Volume I Mechanics e Heat e Electrodynamics SECOND EDITION ABHAY KUMAR SINGH Direct rector Abhay’s I.1.T. Physics Teaching Centre Patna-6 CBS PUBLISHERS & DISTRIBUTORS 4596/1A, 11 DARYAGANJ, NEW DELHI - 110 002 (INDIA) Dedicated to my Teacher Prof. (Dr.) J. Thakur (Department of Physics, Patna University, Patna-4) ISBN : 81-239-0399-5 First Edition : 1995 Reprint: 1997 Second Edition : 1998 Reprint : 2000 Reprint : 2001 Reprint : 2002 Reprint : 2003 Reprint : 2004 Reprint : 2005 Copyright © Author & Publisher Alll rights reserved. No part of this book may. be reproduced or transmitted in any form or by any means, electronic or mechanical, including photocopying, recording, or any information storage and retrieval system without permission, in writing, from the publisher. Published by S.K. Jain for CBS Publishers & Distributors, 4596/1A, 11 Darya Ganj, New Delhi - 110 002 (India) Printed at . India Binding House, Delhi - 110 032 FOREWORD Science, in general, and physics, in particular, have evolved out of man’s quest to know beyond unknowns. Matter, radiation and their mutual interactions are basically studied in physics. Essentially, this is an experimental science. By observing appropriate phenomena in nature one arrives at a set of rules which goes to establish some basic fundamental concepts. Entire physics rests on them. Mere knowledge of them is however not enough. Ability to apply them to real day-to-day problems is required. Prof. Irodov’s book contains one such set of numerical exercises spread over a wide spectrum of physical disciplines. Some of the problems of the book long appeared to be notorious to pose serious challenges to students as well as to their teachers. This book by Prof. Singh on the solutions of problems of Irodov’s book, at the outset, seems to remove the sense of awe which at one time prevailed. Traditionally a difficult exercise to solve continues to draw the attention of concerned persons over a sufficiently long time. Once a logical solution for it becomes available, the difficulties associated with its solutions are forgotten very soon. This statement is not only valid for the solutions of simple physical problems but also to various physical phenomena. Nevertheless, Prof. Singh’s attempt to write a book of this magnitude deserves an all out praise. His ways of solving problems are elegant, straight forward, simple and direct. By writing this book he has definitely contributed to the cause of physics education. A word of advice to its users is however necessary. The solution to a particular problem as given in this book is never to be consulted unless an all out effort in solving it independently has been already made. Only by such judicious uses of this book one would be able to reap better benefits out of it. As a teacher who has taught physics and who has been in touch with physics curricula at 1L.T., Delhi for over thirty years, I earnestly feel that this book will certainly be of benefit to younger students in their formative years. Dr. Dilip Kumar Roy Professor of Physics Indian Institute of Technology, Delhi New Delhi-110016. FOREWORD A. proper understanding of the physical laws and principles that govern nature require solutions of related problems which exemplify the principle in question and leads to a better grasp of the principles involved. It is only through experiments or through solutions of multifarious problem-oriented questions can a student master the intricacies and fall outs of a physical law. According to Ira M. Freeman, professor of physics of the state university of new Jersy at Rutgers and author of “‘physic--principles and Insights” -- “In certain situations mathematical formulation actually promotes intuitive understand- ing....... Sometimes a mathematical formulation is not feasible, so that ordinary language must take the place of mathematics in both roles. However, Mathematics is far more rigorous and its concepts more precise than those of language. Any science that is able to make extensive use of mathematical symbolism and procedures is justly called an exact science’’, LE, Irodov’s problems in General Physics fulfills such a need. This book originally published in Russia contains about 1900 problems on mechanics, thermody- namics, molecular physics, electrodynamics, waves and oscillations, optics, atomic and nuclear physics. The book has survived the test of class room for many years as is evident from its number of reprint editions, which have appeared since the first English edition of 1981, including an Indian Edition at affordable price for Indian students. Abhay Kumar Singh’s present book containing solutions to Dr. I.E. Irodov’s Problems in General Physics is a welcome attempt to develop a student’s problem solving skills. The book should be very useful for the students studying a general course in physics and also in developing their skills to answer questions normally encountered in national level entrance examinations conducted each year by various bodies for admissions to profes- sional colleges in science and technology. B.P. PAL Professor of Physics LLT., Delhi PREFACE TO THE SECOND EDITION Nothing succeeds like success, they say. Now, consequent upon the warm welcome on the part of students and the teaching fraternity this revised and enlarged edition of this volume is before you. In order to make it more up-to-date and viable, a large number of problems have been streamlined with special focus on the complicated and ticklish ones, to cater to the needs of the aspiring students. I extend my deep sense of gratitude to all those who have directly or indirectly engineered the cause of its existing status in the book world. Paina June 1997 Abhay Kumar Singh PREFACE TO THE FIRST EDITION When you invisage to write a book of solutions to problems, one pertinent question crops up in the mind that—why solution! Is this to prove one’s erudition? My only defence against this is that the solution is a challenge to save the scientific man hours by channelizing thoughts in a right direction. The book entitled “Problems in General Physics” authored by LE. Irodov (a noted Russian physicist-and mathematician) contains 1877 intriguing problems divided into six chapters. After the acceptance of my first book “Problems in Physics”, published by Wiley Eastern Limited, I have got the courage to acknowledge the fact that good and honest ultimately win in the market place. This stimulation provided me insight to come up with my second attempt—‘Solutions to I.E. Irodov’s Problems in General Physics.” This first volume encompasses solutions of first three chapters containing 1052 problems. Although a large number of problems can be solved by different methods, I have adopted standard methods and in many of the problems with helping hints for other methods. In the solutions of chapter three, the emf of a cell is represented by & (xi) in contrast to the notation used in figures and in the problem book, due to some printing difficulty. I am thankful to my students Mr. Omprakash, Miss Neera and Miss Punam for their valuable co-operation even in my hard days while authoring the present book. I am also thankful to my younger sister Prof. Ranju Singh, my younger brother Mr. Ratan Kumar Singh, my junior friend Miss Anupama Bharti, other well wishers and friends for their emotional support. At last and above all I am grateful to my Ma and Pappaji for their blessings and encouragement. ABHAY KUMAR SINGH CONTENTS Foreword Preface to the second edition Preface to the first edition PART ONE PHYSICAL FUNDAMENTALS OF MECHANICS Kinematics The Fundamental Equation of Dynamics Laws of Conservation of Energy, Momemtum, and Angular Momentum Universal Gravitation Dynamics of a Solid Body Elastic Deformations of a Solid Body Hydrodynamics Relativistic Mechanics PART TWO THERMODYNAMICS AND MOLECULAR PHYSICS Equation of the Gas State. Processes The first Law of Thermodynamics. Heat Capacity Kinetic theory of Gases. Boltzmann’s Law and Maxwell’s Distribution The Second Law of Thermodynamics. Entropy Liquids. Capillary Effects Phase Transformations Transport Phenomena PART THREE ELECTRODYNAMICS Constant Electric Field in Vacuum Conductors and Dielectrics in an Electric Field Electric Capacitance. Energy of an Electric Field Electric Current Constant Magnetic Field. Magnetics Electromagnetic Induction. Maxwell’s Equations Motion of Charged Particles in Electric and Magnetic Fields vi 1-34 35-65 66-101 102-117 118-143 144-155 156-167 168-183 184-195 196-212 213-226 227-241 242-247 248-256 257-266 267-288 289-305 306-324 325-353 354-379 380-407 408-424 PART ONE PHYSICAL FUNDAMENTALS OF MECHANICS 11 KINEMATICS 11 Let v, be the stream velocity and v’ the velocity of motorboat with respect to water. The motorboat reached point B while going downstream with velocity (v, + v’) and thea returned with velocity (v’ - v,) and passed the raft at point C. Let t be the time for the raft (which flows with stream with velocity v,) to move from point A to C, during which the motorboat moves from A to B and then from B to C. Therefore (vy +v)t-1 1 egy MOAI gene (Wot VN ----- ee) A 8 On solving we get v, = + 1 ¢ 3S 1.2 Let s be the total distance traversed by the point and ¢, the time taken to cover half the distance. Further let 2r be the time to cover the rest half of the distance. s s Therefore zr %h FAN @ s s and Zr Otw)e or Be (2) Hence the sought average velocity s 2m (4 +%) s Po T+” S/2y, IS + VD) tM, +2V, 13 As the car starts from rest and finally comes to a stop, and the rate of acceleration and deceleration are equal, the distances as well as the times taken are same in these phases of motion. Let At be the time for which the car moves uniformly. Then the acceleration / deceleration t-At time is each. So, » 14 A @Aty] | At oore afhw 4 i 3 At or Are 2- ASE w Hence Aw cV 1-482 no 55, we (a) Sought average velocity ep = Sa 20cm t” 20s =10cm/s (b) For the maximum velocity, & should be maximum, From the figure & is maximum for all points on the line ‘ac, thus the sought maximum velocity becomes average velocity for the line ac and is equal to : be _ 100m ab 4s =25 cm/s (©) Time fy should be such that corresponding to it the slope & should pass through the point O (origin), to satisfy the relationship se +. From figure the tangent at point d 0 passes through the origin and thus corresponding time f= fy = 16s. 15 Let the particles collide at the point A (Fig.), whose position vector is 7; (say). If t be the 16 time taken by each particle to reach at point A, from triangle law of vector addition : Fa reemten rave pattem y+, 3, 4 so, -F=@G-H)e wo 4 Ws 7-7 p= acral Iy.-%] From Eqs. (1) and (2) part (em cal ar my cal eee N= HF 0 > s o x na" Ya- Vy A ‘ . . [> Which is the sought relationship 1 therefore, 2) iF = x oo, = - > In-nl lv-¥ ‘We shave vem ware @ From the vector diagram [of Eq. (1)] and using properties of triangle 17 v= Vidar +2 vyvcos@ = 39.7km/br (2) v v vsin@ wd Gages)” and % SHOT TY or @= sin? (SP Using (2) and putting the values of v and d % ‘0 O= 19.1° Let one of the swimmer (say 1) cross the river along AB, which is obviously the shortest path. Time taken to cross the river by the swimmer 1. he , (where AB = d is the width of the river) @ veo For the other swimmer (say 2), which follows the quickest path, the time taken to cross the river. Q) c = _ Sd _=> = _ In the time f, drifting of the swimmer 2, becomes Yo .. x= vols 374, (using Eq. 2) @) If £, be the time for swimmer 2 to walk the distance x to come from C toB (Fig.), then x_ Yd . ete y (using Eq. 3) (4) According to the problem f, = f +13 d vd or, te 4, ° v ap vi vu On solving we get Yo 18 Let! be the distance covered by the boat A along the river as well as by the boat B acrc the river. Let vp be the stream velocity and v’ the velocity of each boat with respect water. Therefore time taken by the boat A in its journey 1 1 ~~ UL, A” Tay Vv t 21 and for the boat B 3: Set ese r the boat ty ae Tae at t , Hen 4. HY. Th (wheren=% oes tp Vvt-e Vai ay On substitution by/ty= 18 19 Let vp be the stream velocity and v’ the velocity of boat with respect to water. A VG ; = 1 = 2>0, some drifting of boat is inevitable. Let ¥” make an angle @ with flow direction. (Fig.), then the time taken to cross the rive t= v £ 0 (where d is the width of the river) In this time intezval, the drifting of the boat x= (V' cos 0+ %9)t = (7008 8+ %) Sg = (cot 8 +n coser 0) d A > For init itis %nin (tainimum drifting) i — d - _ Zo (cot 0 +n cosec 0) = 0, which yields y! > 11 v, — cos 0 = -—= -= 0 n 2 x Hence, O= 120° 0 1.10 The solution of this problem becomes simple in the frame attached with one of the bodies. Let the body thrown straight up be 1 and the other body be 2, then for the body 1 in the frame of 2 from the kinematic equation for constant acceleration : ao Bon Bat +5 a? So, 7ha= Vout (because Wy. = 0 and Fy) = 0) or, IFinl= l¥ounlt @) But |¥ol= [%al= ¥% So, from properties of triangle vane Vibe h Hence, the sought distance [Finl= vo V 2(1-sin 8) t= 22m. ¥9 Vo cos (x1/2 11 1.12 5 Let the velocities of the paricles (say ¥;' and 73) becomes mutually perpendicular after time 1. Then their velocitis become A, PAs, OY = We ats y= +e @ As HLT 50, TT'-7'=0 or, (W484) (p+ Bt) = 0 or -yy,+e t= 0 Hence, t= “a => 1s Now form the Eq. 71 = Toa) + Youn? + Pare > = > => Iral= [oy 1 (because here Wi. = 0 and 717) = 0) Hence the sought distance From the symmetry of the problem all the three points are always located at the vertices of equilateral triangles of varying side length and finally meet at the centriod of the initial equilateral triangle whose side length is a, in the sought time interval (say +). wy +¥% - = ~~ Vvr) (as [Foy l= 1 +2) Let us consider an arbitrary equilateral triangie of edge length I (say). ‘Then the rate by which 1 approaches 2, 2 approches 3, and 3 approches 1, becomes : On integrating : 6 1.13 1.14 Let us locate the points A and B at an arbitrary instant of time (Fig.). If A and B are separated by the distance s at this moment, then the points converge or point A approaches B with velocity =4. v—ucos 0 where angle o varies with time. On intergating, 0 Tr ~fas= f(v—ucos a) de, r 0 (where T is the sought time.) T or t= f(v—ucosa) dt a 0 As both A and B cover the same distance in x-direction during the sought time interval, so the other condition which is required, can be obtained by the equation bcm fv,at r So, ut = f'v.cos ode Q) 0 Solving (1) and (2), we get T= wo One can see that if u= v, or u4 a 2 t Thus da= foam 4-5-8 Putting t= 6, Ax=-1 ' 2 a t Similarly 0 fila fo- ade Fn ates Putting t= 6, sy= 2unit For the patd7: w,= 2 and v= -2+2(t-6)= 2(¢-7) v= |v, |= 2(7-2) for 1-7 6 Now, Ax(t)= [2 @-Tdr= 17-1404 48 , Putting t= 4, Arse -1 ‘ Similarly 53) = f2(7-0)de= 141-17-48 Putting t= 7, S521 On the basis of these obtained expressions w, (1), x(t) and s (f) plots can be easily plotted as shown in the figure of answersheet. 10 1.19 1.20 (a) Mean velocity Total_distance covered vp = ace covert Time elapsed s_ aR = 77 7 S0em/s (1) Vo v (b) Modulus of mean velocity vector i f |< = Laz 2Re s2em/s 2) (©) Let the point moves from i to f along the half circle (Fig,) and v and v be the spe at the points respectively. We have 2. , or, v= vo+w,t (as w, is constant, according to the problem) J Go+m,tat So, = 2 sa 0 YotY «xR 2 t Now the modulus of the mean vector of total acceleration Using (4) in (5), we get : Mot bot vty 2 “2 ¢ So, from (1) and (3) 3 (a) we have re So, ve Se ai-208 and We (>) From the equation r= 0, att= 0 and also at r= Arm + So, the sought time Ar = L As v= @1-2a2) 1 a(1-2a9) fores So, ve |= : a(2at-1)) forr> 5 a Hence, the sought distance 12a va s= fvde= f a(1-2ande+ f a@ar-1dr 0 Va Simplifying, we get, s= da (a) As the particle leaves the origin at So, Ace x= fd (ay > t As ae where ¥ is directed towards the +ve x-axis So, Ve= Vo (: 7 3] @) From (1) and (2), t f wo fnft-dare vot-38) ° : Hence x coordinate of the particle at r= 6s. 6 x= 10x6(1- 353) 24cm = 0:24m Similarly at t= 10s 10 x= 10x10 (1-35}= 9 and at t= 20s x= 10x20(1-22.) = ~200em= -2m 2x5 (b) At the moments the particle is at a distance of 10 cm from the origin, x = + 10 cm. Putting x= +10 in Eq. (3) 10= 101(1- ) or, t7-10t+10= 0, So, te r= WE VTE. 5. a5 4 Now putting x= - 10in Eqn (3) t -10= 10(1 35} On solving, t= 52 VB s As t cannot be negative, so, t=(5+Vv35 )s 12 1.22 Hence the particle is at a distance of 10 cm from the origin at three moments of time : t=S2VI5 s,5+v35 s (©) We have v (1-3) So, vo(e-1} fort >t ' So s= foo(t-sa for tx v = vot (1-%r) 0 : : and on J soft-s)arefolt-1) for t>t 2 1 0 : = vot [1 +(1-4)]/2 for t>t (a) 4 4 o-fo(t- ~ f10(0-$) a 400, 0 ° And for t= 8s os 8 t t oof 0(t-s)aefuo(s-s)a 0 s On integrating and simplifying, we get s= 34cm. On the basis of Eqs. (3) and (4), x (0) and s(t) plots can be drawn as shown in the answer sheet. As particle is in unidirectional motion it is directed along the x-axis all the time. As at t=0,x=0 So, Therefore, or, @ As, On integrating, @ 1.23 1.24 13 (b) Let s be the time to cover first s m of the path. From the Eq. s= fvde (using 2) @) According to the problem - yao aVv (as v decreases with time) 4 ri or, -f Wave afas Yo oO «teeroth 2 32 On integrating we get s= —vo 3a ‘Again according to the problem -2. avv or -f- ad v or, Thus @) As So, and therefore 14 1.25 which is Eq. of a parabola, whose graph is shown in the Fig. (b) As FE atizbrj” < dT ee Ve Fe ai 2b1j Q) So, v= Va7(-2bt) = Va7+4b7r" Diff. Eq. (1) wart. time, we get —» dv” ~ we Be 267 So, |W] =w=25 2btj)-(-2bjF (Va? +4747) 2 2bt or, 00S 0. =§ =, Va744b717 O} cosa= YW vw a so, tana= 7 o, as ow" 357] 2bt (d) The mean velocity vector (aiz2bejS de i x ot = ait bry” Hence, [|= Va7+ (=b8)7 = Va7+b717 (a) We have x= atand y= at(l-ar) q@ Hence, y (x) becomes, aX (y_S*)L yy? y (1 ra x- 2x? (parabola) (b) Disferentiating Eq. (1) we get v= @ and v= a(1-2at) Q) 1.26 127 15 So, v= We vy =aV14(1-2af)" Diff. Eq. (2) with respect to time w,= 0 and w= -2aa So, we Vwrt w= 2a0 (©) From Eqs. (2) and (3) ‘We have Ve ajta(1-2ar)j” and We 2aaj” yw _-a(1-2a4)2aa So, cosZ= Le Uw , 4° V2" vw” aVi+(1-2a%) 200 On simplifying. 1-2a= 21 As, fw 0, y= > Differentiating motion law : x= asinwt, y= a(1-coswr), with respect to time, ¥,= @@ cos @t, v,= asin wt So, V= awcosot{+ awsinwtj~ qa) and v= a@= Const. Q Differentiating Eq. (1) with respect to time we 2. -aw7sinatit aw? cos atj @) (a) The distance 5 traversed by the point during the time + is given by s= fd awd aor (using?) . 0 (>) Taking inner product of v” and ao > . 7 2 gi . 2 7 We get, v%n'= (awcoswti+ asin otj) (asin ot (-i)+a0*cos wt -j) So, ¥ We - a? w sin wt cos wt + a” w sin wt cos wt = 0 Thus, 7°L W, ice., the angle between velocity vector and acceleration vector equals 5. Accordiing to the problem . We w(-j) dav, dy, So, wen Gi 0 and w= 3 -w a) Differentiating Eq. of trajectory, y= ax - bx”, with respect to time ay, 248 95, Q 16 1.28 dy dx So, a ‘at| =a je-0 Again differentiating with respect to time 2 2 2 2 d’y ad -20( Zr) -2bx 4% jz=0 dt? dt? dt? 2 dx . or, -w= a)-26( SF) ~2.bx (0) (using 1) dx VX is - or, a7 Vip (sing 1) ¢ cm =a 2D « Hence, the velocity of the particle at the origin =-V (a), (F dy lo Bt a? 3p (vsing Eqns (3) and (4)) Hence, ve Vip (ite) Using (3) in (2) a As the body is under gravity of constant accelration ’, it’s velocity vector and displacemen vectors are: ss ve *% +t a and Ar= r= vp wrte 1 go? @= Oate = 0) (em So, over the first t seconds + Ar rs gt Be ate +e @) Hence from Eq. (3), <¥> over the first ¢ seconds _ 08 = )-8— > The body thrown in air with velocity vy at an angle a from the horizontal lands at point P on the Earth’s surface at same horizontal level (Fig.). The point of projection is taken as origin, so, Av= x and Ay= y (@) From the Eq. Ay= vy t+ 5uyt? O= wsinat 2vp sina & (b) At the maximum height of ascent, v, = 0 As t# 0, so, time of motion t= so, from the Eq. vy =, +2 w, Ay O= (vysina’-2gH vp sin? a Hence maximum height H = During the time of motion the net horizontal displacement or horizontal range, will be obtained by the equation Ace Vogt Sgt vesin 20 8g or, R= vocosat-F (t= yycosat= when R=H 22 22 vosin’ a _ vosin® a or tana= 4, so, a= tan”'4 & 2g (c) For the body, x(¢) and y(t) are X= vocos at (1) 18 1.30 and ye vosinarr— 3 gt? Q) Hence putting the value of ¢ from (1) into (2) we get, 2 2 ye vo sin a(—*—)-1 x = xtana-—£2, vycosa}] 2° |v) cosa 2v) cos” a Which is the sought equation of trajectory i.e. y (x) (4) As the body thrown in air follows a curve, it has some normal acceleration at all the moments of time during it’s motion in air. At the initial point (x = 0,y = 0), from the equation : 2 w, = a (where R is the radius of curvature) % Fi R ve gees am Re (see Fig.) or Ry= At the peak point v, = 0, v= v,= vp cos at and the angential acceleration is zero. Now from the Eq. wn vacos* a ve cos? g- 2, or Re Note : We may use the formula of curvature radius of a trajectory y(x), to solve part (d), 3 2 1+(dy/dx) | @y/ae | We have, v, = v9 cos a, vy = vp sin ot - gt As Vt u, all the moments of time. Thus v= v2-2etvysinatg?t? Now, y - ~E (vgsina-en)= -8z y, Hens ee Now w,=VwWew = e-#5 Ve or w= gs (where v= Vv ¥, 7 y 132 19 As F749, during time of motion On the basis of obtained expressions or facts the sought plots can be drawn as shown in the figure of answer sheet. The ball strikes the inclined plane (Ox) at point O (origin) with velocity vy = V2gh (1) As the ball elastically rebounds, it recalls with same velocity vp, at the same angle from the normal or y axis (Fig.). Let the ball strikes the incline second time at P, which is at a distance J (say) from the point O, along the incline. From the equation ys vy tt 5H? O- voces at— 5 gcosa? where t is the time of motion of ball in air while moving from O to P. 2 Ast 0, 50, t= <> Q) Now from the equation. 1 X= Vy tt Wt? 2 ‘ 1... vsinat+sgsinay 4vysina = —— (using 2) Hence the sought distance, = 4284) sina | gy sina (Using Eq. 1 r ing Eq Total time of motion 2vgsin a 98 oe ina= 22. + ‘ gr sina FE FA @ and horizontal range R__ 5100 85 Rx vcosat or cosa = Faye” ae Q) From Eqs. (1) and (2) (98P 7 | (85) 2 + 22 = (480) (ary On simplifying tt - 2400 +? + 1083750 = 0 20 1.33 1.34 Solving for 1? we get : 2 2400 + V 1425000 2400 + 1194 2 2 Thus t= 4239s = 0-71 min and T= 2455s = 0-41 min depending on the angle a. Let the shells collide at the point P(x, y). If the first shell takes t's to collide with second and At be the time interval between the firings, then X= vy00s 0, = vycos @,(t-A1) (1) and y= vp sin 0, 1-2 gt? y . 1 > = vo sin 0, (¢- At) — 58 (¢- At? @ ly At cos 8, a) From Ea.) t= 9a; @) P From Eqs. (2) and (3) Ye 8 0 2 vo sin (8, - 0,) Mim Tost, rosby * Ate 0 x According to the problem @) 4. vy or dy= vodt y ' Integrating fam vf dt or y= vot @ 0 ° And also we have &. ay or de= aydt= avgtdt (using 1) : t > So, fax an fea, or, x= davgt?= 49 (using 1) 2 2% 0 0 (b) According to the problem vy= vy and v= ay @ So, ve View = Vigeaty? dv ay dy ay Therefi -2. * erefore We ot [ieay tay? dt Tix lay/ovt @ 7 y Diff. Eq. (2) with respect to time. dy, av, dy ain yn 0 and GA = w= a Pa avy , w= [w= avy 1.35 1.36 Hence w,=VwFow? = V avg- —@% — . 2% * 1+ (a/v) Vis (w/ny (@ The velocity vector of the particle v= aitbyj So, Sea: Be From (1) fern afd o, x= at 0 0 And dy = bx dt= batdt , " dob? Integrating faye ao fea on y= Saber From Egs. (2) and (3), we get, y= Be (b) The curvature radius of trajectory y (x) is : 3 1+(dy/ax)? |? | @y/ae | Let us differentiate the path Eq. y= ge with respect to x, toy ang PY. 2% er gr eG From Eqs. (5) and (6), the sought curvature radius : 3 rbot] In accordance with the problem ne w, But wn 2D or vave mds So, vdve= (@°t)ds=a-d7r” or, vdv= ai-d7= ade (because ais directed towards the x-axis) So, foav= afar 0 0 Hence v= 2ax or, v= Vax 21 (1) Q) @) @ 6) 6) 22 1.37 The velocity of the particle v= at So, anna 2.2 And =~. (using v= wo Fe SE (using v= at) From sm fvde ' -2aRny= afvdtm Sar? ° 4ay_ 2? s day? ° a "R From Eqs. (2) and (3) w, = 4nay Hence w= Vw+w? = Va'+ (nan) = aV1+ 160 HY = 08 m/s? 1.38 According to the problem Iw f= Lw,1 For v(d), ef - ¥ Integrating this equation from vo = v< v and Ox fs ¢ ’ ' dv 1 Yo Af #-4fe me 0 ° TR Now for v(s), - ra z , Integrating this equation from vys vs v and Os sss v s dv 1 v Ss ,, ° a Hence ve we (b) The normal acceleration of the point 2 v = 2s/R ¥ e . Wat = RE (using 2) And as accordance with the problem |w,l= [w,] and w, ia, Lw, ve 2 s0, we V2w,= VE ge ME = VEZ @ ® @) 2 1399 1.40 141 23 From the equation v= avs. mn Ste Soe Se oien Sat at” 2Vs dt” 2 wpe ee ES mn” ROR As w, is a positive constant, the speed of the particle increases with time, and the tangential acceleration vector and velocity vector coincides in direction. Hence the angle between V°and Wis equal to between w,iz, an W, and a can be found [wal a? s/R_ 28 by means of the formula : tan a=" = ¥ wl" a2 From the equation I= asinot Be ye awcoswt dt So, w= 4. ~aosinwt, and Q) ¥_ aw cos’ wt we Ge Coe @ (a) At the point J= 0,sinwt= 0 and cos@t= + 1 so, wf= 0, x etc. fw Hence Similarly at/= * a, sinwt= * 1 and cos «t= 0, so, w,= 0 Hence w= [w= ao? As w,= @ and at f= 0, the point is at rest So, (9) and s(t) are, v= at and s= far? () Let R be the curvature radius, then 2 42 ve att 2as,. vgn ee Tw 22 Casing 1) But according to the’problem w, = be* 2,2 2 So, bt*= 7 or, - in (using 1) 2) a R= + br? Therefore w= Vwe+we = Va'+(2as/R)? = V a + (4 bs*/a?)” (using 2) Hence w= aV 1+ (46970) 24 1.42 (a) Let us differentiate twice the path equation y (x) with respect to time. Do rar, PY. 9q/ (42) pd? fa nent, Boas) oa Since the particle moves uniformly, its acceleration at all points of the path is normal and at the point x = 0 it coincides with the direction of derivative d? y/dt?. Keeping in mind =y, that at the point x= 0, @| , We get we =| -2av=w, at a= 0 av wk So, Ww, 2ave For R= 5- Note that we can also calculate it from the formula of problem (1.35 b) (b) Differentiating the equation of the trajectory with respect to time we see that bx By ay Boo ) which implies that the vector (b’xi"+ a’yjJ is normal to the velocity vector ve er, 2 "which, of course, is along the tangent. Thus the former vactor is along the normal and the normal component of acceleration is clearly dx ad . Bx tat ay "(Os ayy onusing w,= wen |n']. Atx= 0, y= * band so atx =0 x=0 Differentiating (1) oi) oS tt) 218) Also from (1) Be oax=0 So 2) = © v (since tangential velocity is constant = v ) om (8-27 dt a’ w and |m| - = This gives R = a*/b. 1.43 1.44 Let us fix the co-ordinate system at the point O as shown in the figure, such that the radius vector 7of point A makes an angle 6 with x axis at the moment shown. Note that the radius vector of the particle A rotates clockwise and we here take line ox as reference line, so in this case obviously the do . angular velocity o= 7 taking anticlockwise sense of angular displacement as positive. Also from the geometry of the triangle OAC R r sin” sin(m—20) %? 77 2R ens Let us write, 7 rcosOj+rsinOj= 2Rcos’0j+ Rsin20j” Differentiating with respect to time, ie or W= 2R2cos 0(- sin 0) 4207428 cos 20 405+ or, a a ©) {sin2 07> xs 207] or, = 2Rw (sin 20i—cos?0j) So, |v] or v= 20R=0-4m/s. As @ is constant, v is also constant and w, = e. 0, vy? _ (2oRY R So, we we = 407R= 0:32 m/s? Alternate : From the Fig. the angular velocity of the point A, with respect to centre of the circle C becomes d(26) dt Thus we have the problem of finding the velocity and acceleration of a particle moving along a circle of radius R with constant angular velocity 2 w. - 2(=gr)-20- constant Hence v= 2oR and 2 (2mRP we ORY 402 wane R 40?R Differentiating @ (t) with respect to time #2 = 0,=2at (1) For fixed axis rotation, the speed of the point A: y veoR= 2atR or R= > (2) 26 1.45 1.46 Differentiating with respect to time w= #. 2aR~ ~, (using 1) But “ORO Waar 24t” (using 2) So, wa Vwlew? =V(v/ty+(2atvy 2Vie4a71 The shell acquires a constant angular acceleration at the same time as it accelerates linearly. The two are related by (assuming both are constant) w, Bb 1 2nn Where w= linear acceleration and 6 =_angular acceleration Then, o- V2B2xn- 25 (2any But v?= 2w/, hence finally _ nny 7 Let us take the rotation axis as z-axis whose positive direction is associated with the positive direction of the cordinate @, the rotation angle, in accordance with the right-hand screw rule (Fig.) (a) Defferentiating ¢() with respect to time. Z $n g- 3bt7= 0, (1) and ao do, oe = -6bt (2) di Q) From (1) the solid comes to stop at At= t= Ve va OS The angular velocity w= a-3bt?, for 0st V0/3b fea “Pe. 3bP)dt So, < @ >= tom fat bt ays 5 are = 20/3 vapt dt Similarly B= [B,|= 65¢ for all values of ¢. 1.47 1.48 27 Va 6bt dt So, = Bar = V3ab far” af fae (b) From Eq. (2) B,= - 6b¢ So, (B,),= Va73b = - 6b 3B = -2Vab Hence B= | (82)... vazap | 7 2V3ab Angle a is related with |w,| and w, by means of the fomula : tana= where w,= @7R and |w,|= BR qa Wa II’ where R is the radius of the circle which an arbitrary point of the body circumscribes. From the given equation B = de, at (here B= ge » as B is positive for all values of 1) t Integrating within the limit [de> = a fede or, onsar 2 2 So, me oR» () re SR and Iw, |= BR= atR Putting the values of | w,| and w, in Eq. (1), we get, @t*R/4_ at? 4 me 2B #0 oe, co [ (Snel 3 In accordance with the problem, 6, < 0 Thus -2. k Vo, where k is proportionality constant kt or, - 2-k fae or, Vo = vay ~ SE (Q 2vaq When @ = 0, total time of rotation t= t= k 28 1.49 1.50 2Veng/k 2,2 f (one Ayo-#evou at 1 foar ‘0 Average angular velocity < @ > = ——— = ————___—— a 2Ve /k peal ee ¥@ verse < 0 > [ager 7) 7% # | [2% - 03 We have w= Wy - ap = 2 Integratin this Eq. within its limit for (p) ¢ 9. : k father farm Pkt 0 0 Op Hence Bae") @ (b) From the Eq., @ = wy k@ and Eq, (1) or by differentiating Eq. (1) vke W= We Let us choose the positive direction of z-axis (stationary rotation axis) along the vector B,, In accordance with the equation do, at or 0s 8, z or, w,da,= B,dp= Bcos pd g, Wz Integrating this Eq. within its limit for @, (9) » x ° 0 feo,= Bof cos ode gy ° 0 or, Hence, = V2 By sing The plot w, (¢) is shown in the Fig. It can be seen that as the angle @ grows, the vector G@ first increases, coinciding with the direction of the vector By (w, > 0), reaches the maximum at p= 9/2, then starts decreasing and finally turns into zero at @ = x. After that the body starts rotating in the opposite direction in a similar fashion (w, < 0). As a result, the body will oscillate about the position p= @/2 with an amplitude equal to x/2. 29 1.51 Rotating disc moves along the x-axis, in plane motion in x -y plane. Plane motion of a solid can be imagined to be in pure rotation about a point (say /) at a certain instant known as instantaneous centre of rotation. The instantaneous axis whose positive sense is directed along @ of the solid and which passes through the point /, is known as instantaneous axis of rotation. Therefore the velocity vector of an arbitrary point (P) of the solid can be represented as : vps Ox rp @) On the basis of Eq. (1) for the C. M. (C) of the disc y en Ox7y @ According to the problem v7tti and Ott ie. SLx-y plane, s0, to satisy the Eqn. (2) 7¢;is directed along (-j ). Hence point jo we Tis at a distance ro, = y, above the centre of the disc along y-axis. Using all these facts in Eq. (2), we get Ve Yom ay or y= @) (a) From the angular kinematical equation @, = Og + Bt 4) o= BL On the other hand x= vt, (where x is the x coordinate of the C.M.) x or, ree 6) From Eqs. (4) and (5), © = v - Y, Using this value of cw in Eq. (3) we get y= = im 7” 4 (hyperbola ) (b) As centre C moves with constant acceleration w, with zero initial velocity So, xedwi? and v,= wt Therefore, ve wV% =v2xw y we Hence ya “_ V2" (parabola) @ 30 152 1.53 The plane motion of a solid can be imagined as the combination of translation of the C.M. and rotation about ch M. So, we may write co vote = +OxKe (1) and Wat Wot Wao Were (—MRe)+(Bx%c) ©) Tac is the position of vector of A with respect to Cs In the problem vo = v= constant, and the rolling is without slipping ic., ves v= oR, So, We = 0 and B= 0. Using these conditions in Eq. (2) a - > nyt a Wan O7(=F4c)= O7R(~ ic) = (tae) Here, it, ¢ is the unit vector directed along tc. 2 > a Hence w,= "> and Ww, is directed along (~ il, -) or directed toward the centre of the wheel. (b) Let the centre of the wheel move toward right (positive x-axis) then for pure tolling on the rigid horizontal surface, wheel wil have to rotate in clockwise sense. If « be the angular velocity of the wheel then w = ee x Let the point A touches the horizontal surface at t= 0, further let us locate the point A att= 4, When it makes @ = wf at the centre of the wheel. From Eqn. (1) Was Ver Ox igo = vit w(-&) x [Roos 0 (-j) +R sin 8 (-i)] or, w= vit oR[coswt(-i)+sinotj } = (v- cos wt)it+vsinwtj (asv= wR) So, v= V (v-vcos ax)" + (v sin wt)? = vV2(1-cos wf) = 2 vsin (wt /2) Hence distance covered by the point A during T= 2./w 2x/o a= frdta fav sinoe/2) ar 8 88 0 Let us fix the co-ordinate axis xyz as shown in the fig. As the ball rolls without slipping along the rigid surface so, on the basis of the solution of problem 1.52 : Ha + Sxr= 0 i} Thos v= oR and Gtt(-k) as thi @) 31 and G+ Br = 0 4 i w.= BR and Btt (-k) as wt A VetWR=Vs At the position corresponding to that of Fig., in accordance with the problem, B W,= W, SO v= Wt ¥ \ Ye _ wt wee { «£ * and om RnR md B= F (using 1) 4 Vs (a) Let us fix the co-ordinate system with the frame = /777777 attached with the rigid surface as shown in the Fig. As point O is the instantaneous centre of rotation of the ball at the moment shown in Fig. 50, v= 0, Now, ihe Weak, = voit @(-R)XRGYS= (Vet OR) So, Uys 2veim 2wei (using 1) Similalry ¥p= ¥24 Tx Foo voit w(-k)xR GD) = vit OR(~j)= Veit Ve(-]) So, vg =VZ v,=VZ wt and Vf is at an angle 45° from both Zand j (Fig.) ©) ym We +0" (7) + BX Fc 2, A_W #8 = 0 Fe) = FE Woe ) (using 1) where ligg is the unit vector along Foc \ 80, Wo= 2. fe (using 2) and wy is W directed towards the centre of the ball Be Now Wy = Wo +0? (- ac) + BX Tac be 0 y = wit aR (-j)+B(-E)xRF_ a oe ew = = (w+ BR)i+ FR (-J) (using 1) = 2witR-Cd) ae De So, wma V awa =2w 1+(5] 2R Similarly Wy = Wy + 0 (- 750) + BK Tac = wit oR Ci} +B XR] = |v S]F RCA (osing 1) 32 - (» - 7 T+ wf (using 2) ap wey ae 1.54 Let us draw the kinematical diagram of the rolling cylinder on the basis of the solutios of problem 1.53. A Vaz2\p _WetBr \ CA We So, Wy Br ve=ViVe As, an arbitrary point of the cylinder follows a curve, its normal acceleration and radius of curvature are related by the well known equation y= > R , % so, for point A, ay 7 av or, Ry= = 4r (because v,= wr, for pure rolling) Similarly for point B, % “ao” Ry (V2 v, )* wrens 45° = W2%e na } , vw or, Ry= 2V2—= 2V2r or 1.55 The angular velocity is a vector as infinitesimal rotation commute. Then the relative angular velocity of the body 1 with respect to the body 2 is clearly. B=, -B, as for relative linear velocity. The relative acceleration of 1 w.t.t. 2 is (2), 1.56 157 33 where S' is a frame corotating with the second body and S is a space fixed frame with origin coinciding with the point of intersection of the two axes, , dl, (de) ut dt I dt JF Orem . > do, Since S’ rotates with angular velocity W,, . However (2) = 0 as the first body rotates s with constant angular velocity in space, thus Bae a xa, Note that for any vector 5? the relation in space forced frame (k) and a frame (&’) rotating with angular velocity @ is d d —. ap] “| +x K xK Wehave = atitbr?j- q So, o= Vay + (67), thus, Ol;. 10. = 7.81 rad/s Differentiating Eq. (1) with respect to time B 2. aTh2bj Q) So, Ba Vas (2b and Bl= 105 * 13 rad/s? © cosan 2B, (ois b7j) ois 27) op (at? + (bry Vas by (2b Putting the values of (a) and (6) and’ taking ¢ = 10s, we get a= 17° (a) Let the axis of the cone (OC) rotates in anticlockwise sense with constant angular velocity @” and the cone itself about it’s own axis (OC) in clockwise sense with angular velocity @, (Fig.). Then the resultant angular velocity of the cone. Te T+ @ As the rolling is pure the magnitudes of the vectors @ and ay can be easily found from Fig. oy o'= Roig? Mom WR Q) As @ 1G, from Eq. (1) and (2) 34 1.58 o- Vo var v 2 'y\2 v (rare] +) ” Reosa™ 23 md/s (b) Vector of angular acceleration S d@ +a, ~ B= 2. feo) (as @” = constant.) The vector @, which rotates about the OO’ axis with the angular velocity @, retains i magnitude. This increment in the time interval dt is equal to |d@| = we w’ de or in vector form diy = (GW x By) de. Thus Be xa G The magnitude of the vector Bis equal to B= o' a (as @ 105) viv ReotaR So, Suna- 2:3 rad/s The axis AB acquired the angular velocity w= Be (a) 0 Using the facts of the solution of 1.57, the > angular velocity of the body a =BeE o- Varro” = Vg + Bat? = 0-6 rad/s And the angular acceleration. pe dat a@' +) dw doy a dt a ds dB ~ But “G7 8 xd, and 7 Bor So, B= (Box )+Bo As, BL, so, B= V (0% By) + BA = By V1 + (yt) = 0-2 rad/s? 35 2. THE FUNDAMENTAL EQUATION OF DYNAMICS 159 Let R be the constant upward thurst on the acrostat of mass m, coming down with a 1.61 2 constant acceleration w. Applying Newton’s second law of motion for the acrostat in projection form F,= mw, mg-R= mw qa Now, if Am be the mass, to be dumped, then using the Eq. F,= mw, R-(m-Am)g= (m-Am)w, oy 2mw From Eqs. (1) and (2), we get, Am= 2 oT Let us write the fundamental equation of dynamics for all the three blocks in terms of projections, having taken the positive direction of x and y axes as shown in Fig; and using the fact that kinematical relation between the accelerations is such that the blocks move with same value of acceleration (say w) mg 8-T,= mw Q) T, -T, - km g=m,w @ and T,- kn, g= m,w @) The simultaneous solution of Eqs. (1), (2) and (3) yields, [img - k(m, +m.) ] | —mimim ¢ (1+k) mp and q, m8 ‘Mog As the block my moves down with acceleration w, so in vector form we [tg - K (m, +m) 18" Mgt my +m, Let us indicate the positive direction of x-axis along the incline (Fig). Figures show the force diagram for the blocks. Let, R be the force of interaction between the bars and they are obviously sliding down with the same constant acceleration w. mg + my, +m, 36 1.62 Newton’s second law of motion in projection form along x-axis for the blocks gives : m, gsin a — km, gcosa+R= m,w qQ) m, g sin a - R - km, g cos a= mw Q) Solving Eqs. (1) and (2) simultaneously, we get ym + ham | my +m my m, (k, - k,) g cos o m, +m, (b) when the blocks just slide down the plane, w= 0, so from Eqn. (3) kmthm 4 m, +m, oF, (my + my) sina = (k, m, + km.) cos ~ &mtem) mm, +m, Case 1. When the body is launched up : Let k be the coefficeint of friction, u the velocity of projection and / the distance traversed along the incline. Retarding force on the block = mg sin a +kmg cos a. and hence the retardation = gsin a +kg cos a. Using the equation of particle kinematics along the incline, we gsin a g cos nd R= 6) gsina-gcosa Hence tana O= w?-2(gsina+kgcosa)! 2 o '" T@iinatkgesa) @ and O= u-(gsina+kgcosa)t or, u= (gsina+kgcosa)t (2) Using (2) in (1) J= 3 sina + kg cos 0) 1? @) Case (2). When the block comes downward, the net force on the body = mg sino - kam g cos 0. and hence its acceleration = g sino ~ kg cos & Let, t be the time required then, t= E @sina—kg.cosa)1? @ From Eqs. (3) and (4) P| sina-keos #2? sina+kcosa But5= a (according to the question), Hence on solving we get 2 k= MY una= 016 (n° +1) 163 1.64 165 37 At the initial moment, obviously the tension in the thread connecting m, and m, equals the weight of m,. (@) For the block m, to come down or the block m, to go up, the conditions is m,g-Tz0 and T-m,gsina-frz0 where T is tension and f, is friction which in the limiting case equals km,g cosa. Then or =m, g- my sina. > km, g cos 0 or 72, (k cos 0+ sin a) m (b) Similarly in the case m, g sin o- m2 8> frig Of, m, g sin & —m,g>km,g cosa or, 7 < (sin a - kos a) m (©) For this case, neither kind of motion is possible, and fr need not be limiting. Hence, (kos 0+ sin a) > > (sin a — bcos a) 1 From the conditions, obtained in the previous problem, first we will check whether the mass m, goes up or down. Here, m,/m, = 17> sin ot + k cos ot, (substituting the values). Hence the mass m, will come down with an acceleration (say w). From the free body diagram of previous problem, m,-g-T= mw @ and T-m,gsino.- km, g cos a= m,w 2) Adding (1) and (2), we get, my gm, g sina. - km, g cos. a= (m, +m) w (m,/m,-sina-keosa)g (y-sina-kcosa)g ” (14+ m/m,) 1+n Substituting all the values, w= 0-048 g ~ 0-05 g ‘As m, moves down with acceleration of magnitude w= 0.05 g > 0, thus in vector form acceleration of m, : w- Masina— kes g.95 2° 1+y Let us write the Newton’s second law in projection form along positive x-axis for the plank and the bar fre mw, fre mw, @ 38 1.66 1.67 At the initial moment, fr represents the static friction, and as the force F grows so does the friction force fr, but up to it’s limiting value ie. fr= frgmaxy= KN= km g. F Fr. Unless this valuc is reached, both bodies moves Fp as a single body with equal acceleration. But as soon as the force fr reaches the limit, the bar starts sliding over the plank i.e. w,2 W. Substituting here the values of w, and w, taken from Eq. (1) and taking into account that ting, 1 "2 f,= km, g,we obtain, (at - kan, g)/m, = — g, were the sign "=" corresponds to the moment ’ my h t= Ip (Say) k Heace ton 8am) A If tf, then w, = me (constant). and 1 Ww, = (at - km, g)/m, On this basis w, (t) and w,(t), plots are as shown in the figure of answersheet. Let us designate the x-axis (Fig.) and apply F,= mw, for body A : mgsina-kmgcosa= mw or, w= gsina-kgcosa Now, from kinematical equation : Isec a = 0+ (1/2) wt? or, t= V2Tsec a/(ein a — F008 0) g N = V21/ (sin 2 a/2 — kcos" a) g LV (using Eq. (1)). a( Mp2 kext) for tain » ——~n 0 A ti ie. 20826 2 bens asin a= 0 1 ° or, tn2a=-7>a= 49 k and putting the values of o, k and / in Eq. (2) We get fain = 1s. Let us fix the x -y co-ordinate system to the wedge, taking the x-axis up, along the incline and the y-axis perpendicular to it (Fig.). 1.68 39 Now, we draw the free body diagram for the bar. Let us apply Newton’s second law in projection form along x and y axis for the bar : Tcos B-mgsina-fr= 0 q@ TsinB +N-mgcosa= 0 or, N= mgcosa-TsinB 2 But f, = KN and using (2) in (1), we get T= mgsin a+ kmg cos a/(cos B + ksin B) @ For Try, the value of (cos B + ksin B) should be maximum d (cos B + ksin 8) . . So, ap O or tnfp=k Putting this value of B in Eq. (3) we get, 7. = —mglsina+keosa) __ mg (sina + kos a) MPa Vik? eV Viek? First of all let us draw the free body diagram for the small body of mass m and indicate x-axis along the horizontal plane and y - axis, perpendicular to it, as shown in the figure. Let the block breaks off the plane at t= % ie. N= 0 So, N= mg-atysina=0 o, ye te @ From F, = mw,, for the body under investigation : md y/dt= atcos.o. ; Integrating within the limits for v(t) mf de acona fet (using Eq. 1) 0 0 ds_ acosa 2 So, oS @) Integrating, Eqn. (2) for s(t) 3 acosat se @) Using the value of f= fy from Eq. (1), into Eqs. (2) and (3) mg cosa and _me cos Oo ve 2asin’a 6a’ sin? a 40 1.69 Newton’s second law of motion in projection form, along horizontal or x — axis i.e. 1.70 LIL F, = mw, gives. F cos (as)= mv (as a= as) as on Fos (as) ds= mv dy Integrating, over the limits for v (s) “ 2 reo (a) d= 2F or ve V ma = V2gsina/3a (using F= 7%) which is the sought relationship. From the Newton’s second law in projection from : For the bar, T-2kmg = (2m) w For the motor, T- kong = mw’ Now, from the equation of kinematics in the frame of bar or motor : 1 2 I= 50vew)e From (1), (2) and (3) we get on eliminating T and w/ t= Vike +3) @ Q) @) Let us write Newton’s second law in vector from F'= m/w, for both the blocks (in the frame of ground). T+ m, z= mW ® T+m,g= mv, @ These two equations contain three unknown quantities W,, W;, and T. The third equation is provided by the kinematic relationship between the accelerations : ee eewy We ee, he Rw Q) where Ww" is th acceleration of the mass my with respect to the pulley or elevator car. Summing up termwise the left hand and the right-hand sides of these kinematical equations, we get 172 41 R+w- 2m, 4) The simultaneous solution of Eqs-(1), (2) and (4) yields ap __ (my ~ my) B+ 2m hy we See eo m,,m, Using this result in Eq. (3), we get, 2mm, m,+m, (%- 8) mam git) m, +m, ° Using the results in Eq. (3) we get W = (b) obviously the force exerted by the pulley on the celing of the car > zw 4mm Pe -27s ei) Note : one could also solve this problem in the frame of elevator car. Let us write Newton’s second law for both, bar 1 and body 2 in terms of projection having taken the positive direction of x, and x, as shown in the figure and assuming that body 2 starts sliding, say, upward along the incline T,-m, gsina= m,», Q a N m,8-T,= m,w 2 Ti For the pulley, moving in vertical direction T from the equation F,= mw, 2T-T,= (m,)w,= 0 fo mg h | (as mass of the pulley m, = 0) 2 vx2 or T,= 27, 3) As the length of the threads are constant, the m9. kinematical relationship of accelerations becomes w= 2m, (4) Simultaneous solutions of all these equations yields : 28 ( 2 7 -sina ) 1 _ 2g (2y-sina) mm, anel («z ) @n+) m, As 1> 1, w is directed vertically downward, and hence in vector form w= 42 1.73 1.74 Let us write Newton’s second law for masses m, and m, and moving pully in vertical direction along positive x - axis (Fig.) : myg-T= mW © yy = mr m,g-T= mW, 2 T,-2T= O(asm= 0) or T=2T @) Again using Newton’s second law in projection form for mass m, along positive x, direction Fig.), we get T, = my Wo (4) The kinematical relationship between the accelerations of masses gives in terms of projection on the x - axis Wy, + Wo, = 2 (5) Simultaneous solution of the obtained five equations yields : we [4 m, m, + mo (m, - m.) 18 1° 4 my my + Mg (mm; + ma) In vector form xe _ (Amy my + mo (my - m) 1" We ee 4m, m, +m, (m, +m) As the thread is not tied with m, so if there Were no friction between the thread and the ball m, the tension in the thread would be zero and as a result both bodies will have free fall motion. Obviously in the given problem it is the friction force exerted by the ball on the thread, which becomes the tension in the thread. From the condition or language of the problem wy,>w,, and as both are directed downward so, relative acceleration of M = Wy,-w,, and is directed downward. Kinematical equation for the ball in the frame of rod in projection form along upward direction gives : 1 Is 5 Wy — Wy)? w Newton’s second law in projection form along T=f vertically down direction for both, rod and ball =jP gives, Mg -fr= Mwy Q) fr nm mg -fr= mv ® | w Multiplying Eq. (2) by m and Eq. (3) by M m and then subtracting Eq. (3) from (2) and after ¢ using Eq, (1) we get 1 21Mm M, Ope my a ‘t 175 1.76 43 ‘Suppose, the ball goes up with accleration w, and the rod comes down with the acceleration w. As the length of the thread is constant, 2, = w qa) From Newton’s second law in projection form along vertically upward for the ball and vertically downward for the rod respectively gives, T-mg= mw, Q) and Mg-T'= Mw, Q) but T=2T (because pulley is massless) 4 From Eqs. (1), (2), (3) and (4) w= - Gtems, eae (in upward direction) and wy = aaah (downwards) From kinematical equation in projection form, we get t= Loewe? as, w, and Ww, are in the opposite direction, Putting the values of w, and w,, the sought time becomes t= V21)+4)/32-weg = 14s Using Newton’s second law in projection form along x-axis for the body 1 and along negative x-axis for the body 2 respectively, we get m,g-T,= m,™ qa) T,-m, 8 = mW, 2 For the pulley lowering in downward direction from Newton’s law along x axis, —2T= 0 (as pulley is mass less) or, T,= 27, (3) As the length of the thread is constant so, wy= 2 (4) The simultaneous solution of above equations yields, nye GPE | BAD tag Fa wy ®) Obviously during the time interval in which the body 1 comes to the horizontal floor covering the distance h, the body 2 moves upward the distance 2h. At the moment when the body 2 is at the height 2h from the floor its velocity is given by the expression : B= 2w, (2m) = 2/ 20-28) 9, . SAO =2) ne4 n+4 After the body m, touches the floor the thread becomes slack or the tension in the thread zero, thus as a result body 2 is only under gravity for it’s subsequent motion. ad LH 1.78 Owing to the velocity v, at that moment or at the height 2h from the floor, the body 2 further goes up under gravity by the distance, %_ 4n-2) 2g n+4 Thus the sought maximum height attained by the body 2 : He 2heh = 2h M=2_ oh +4) ned Let us draw free body diagram of each body, i.e. of rod A and of wedge B and also draw the kinemetical diagram for accelerations, after analysing the directions of motion of Aand B. Kinematical relationship of accelarations is : , tana= 4+ @) We Let us write Newton’s second law for both bodies in terms of projections having taken positive directions of y and x axes as shown in the figure. m,§-N cos a= my Wy 2) and Nsin a= mg Wy @) Simultaneous solution of (1), (2) and (3) yields : m, gsin a Wy oe an m, Sin 0. + my Cot a.cos a” (1+ col” a) Wee vA tana” (tana+7 cota) AB — Wy > We Note : We may also solve this problem using conservation of mechanical energy instead of Newton's second law. Let us draw free body diagram of each body and fix the coordinate system, as shown in the figure. After analysing the motion of M and m on the basis of force diagrams, let us draw the kinematical diagram for accelerations (Fig.). As the length of threads are constant so, dsyyp~ sy and as ny and Vy, do not change their directions that why |Faw| = [he | = w Gay) and Wrag tt Vig and Wy tt iy 1.79 > Wn rT Tr T fe N Not Ny Om 7 > Wn Tom =x "4 = Wa AS Hh = Why + hy so, from the triangle law of vector addition Wy = V2 @ From the Eq. F,= mw, , for the wedge and block : T-N= Mw, Q) and N= mw @) Now, from the Eq. F,= mw, , for the block mg-T-kN= mw 4) Simultaneous solution of Eqs. (2), (3) and (4) yields : my we ——™___.___& ___ (lan+2m+M) (k+2+M/m) Hence using Eq. (1) wow 82 m™” (2+k+M/m) Bodies 1 and 2 will remain at rest with repect to bar A for Wain SWS Winagy WHETE Wyagi the sought minimum acceleration of the bar. Beyond these limits there will be a relative motion between bar and the bodies. For 0< ws W.i.. the tendency of body 1 in relation to the bar A is to move towards right and is in the opposite sense for W= Waa. On the basis of above argument the static friction on 2 by A is directed upward and on 1 by A is directed towards left for the purpose of calculating Win. Let us write Newton’s second law for bodies 1 and 2 in terms of projection along positive x-axis (Fig.). T-fry= mw or, fr»= T-mw (1) N,= mw Q) As body 2 has no acceleration in vertical direction, so fry= mg-T (3) From (1) and (3) (fr, + fra) = m@-w) (4) But fry + fry sk (N, +N) or fry + fr, sk (mg + mw) 6) 46 1.80 1.81 From (4) and (5) m(G-w) k the plane with respect to 1, say with acceleration w,,, then, w.= Ww», + W, (Fig.) Let us write Newton’s second law for both bodies in projection form along positive yp and x, axes as shown in the Fig. mz B 0S. —N = mz Woy, = mal warn * Min) ]™ ma[ 04m sina] or, m,g.cosa-N= m,w, sina @) and Nsina = mw, Q) Solving (1) and (2), we get m, g sin cos gsin a.cos @ m,+m,sin?a — (m,/m,) +sin?a Wi 47 1.82 To analyse the kinematic relations between the bodies, sketch the force diagram of each 1.83 body as shown in the figure. On the basis of force diagram, it is obvious that the wedge M will move towards right and the block will move down along the wedge. As the length of the thread is constant, the distance travelled by the block on the wedge must be equal to the distance travelled by the wedge on the floor. Hence ds,,y = dS AS Vpy and ¥,, do not change their directions and acceleration that’s Why Wry tt Vay and Wy tt Vy and Wyyy = Wy = W (say) and accordingly the diagram of kinematical dependence is shown in figure. Wr AS Wi," Wy yy + Wy, 80 from triangle law of vector addition. Wy = Viet Wey — 2 Wy ye Wy COS = WV2I - cos a) @) From F,= mw, (for the wedge), T= Tcosa+Nsina= Mw @) For the bar m let us fix (xy) coordinate system in the frame of ground Newton’s law in projection form along x and y axes (Fig.) gives mg sina - T= m Wc [Wa gt Mare] = [aac * Map C08 (— a)] = mw (1 — cosa) 8) mgcosa—N= mwqiy)™ ml Wmancy)* u(y] = LO wsin a] (4) Solving the above Eqs. simultaneously, we get we —_mgsina M+2m(1-cosa) Note : We can study the motion of the block m in the frame of wedge also, alternately we may solve this problem using conservation of mechanical energy. Let us sketch the diagram for the motion of the particle of mass m along the circle of radius R and indicate x and y axis, as shown in the figure. (a) For the particle, change in momentum A p= mv (-i)-mv(@j) so, | Ap |= V2 mv and time taken in describing quarter of the circle, 48 1.84 1.85 aR Are 38 o> 2 Hence, = 1821, Em 2Emv? = _, fA At" xR/2v" aR ve > (b) In this case VE Pix 0 and p= mw,t(~-i), so. |Ap|=mw,t Hence, || = 121. my, While moving in a loop, normal reaction exerted by the flyer on the loop at different points and uncompensated weight if any contribute to the weight of flyer at those points. (a) When the aircraft is at the lowermost point, Newton’s second law of motion in projection form F, = mw, gives N-mg= or, N= mg+ (b) When it is at the upper most point, again from F, = mw, we get Nome = ” my? N =mg= 0-7kN (c) When the aircraft is at the middle point of the loop, again from F, = mw, my? N= = 14kN The uncompensated weight is mg. Thus effective weight = VN + m” g” = 156 KN acts obliquely. Let us depict the forces acting on the small sphere m, (at an arbitrary position when the thread makes an angle 0 from the vertical) and write equation F = mw via projection on the unit vectors iu, and i, From F,= mw,, we have mg sin 0 = m &% 6 at vdv vdv =m. ds "1(-d0) (as vertical is refrence line of angular position) 49 or vdve= ~glsind dd Integrating both the sides : ° dv= -gif sinddo J, 2 y or, 5 = 8100s 0 2 Hence “= 2.gcos = w, a) (Eq. (1) can be easily obtained by the conservation of mechanical energy). From F, = mw, T-mgcos0 Using (1) we have T= 3 mg cos 0 @ Again from the Eq. F,= mw,: ™ mg sinO= mw, or w,= gsin® @) Hence w= Vw2+we = V(gsin0)?+(2g cos 0)? (using 1 and 3) = gV1+43c0s70 (b) Vertical component of velocity, v, = vsin @ So, v2 = v?sin? 0 = 2g 1cos @ sin? @ (using 1) in? For maximum v, or vy, ean O sin 9) 0 hich yield 0s 0 = + which yields c w Therefore from (2) T= amg Fe V3 mg (©) We have W= w,i1, +, ll, thus w= Wy) + Way) But in accordance with the problem w, = 0 So, Wiyy + Way = or, gin 0 sin @ + 2g cos 0 (- cos 8) = 0 on cosO= 4 or, 0= 547° v3 50 1.86 The ball has only normal acceleration at the lowest position and only tangential acceleration at any of the extreme position. Let v be the speed of the ball at its lowest position and / be the length of the thread, then according to the problem 2 ¥ . Te gsina (1) where ot is the maximum deflection angle From Newton’s law in projection form : F,= mw, ~mgsin 0 = my 4% 140 or, -glsinOdO= vdv On integrating both the sides within their limits. a 0 -alf sinodo~ f vav 0 ¥ or, v? = 2gl (1-cos a) Q) Note : Eq. (2) can easily be obtained by the conservation of mechanical energy of the ball in the uniform field of gravity. From Eqs. (1) and (2) with @ = a 2gl (1 ~ cos a) = Ig cos a or, cosa= 5 S0, a= 53° Let us depict the fgrces acting on the body A,(Which are the force of gravity mB"and the normal reaction N ) and write equation F = mw via projection on the unit vectors u, and u, Fig.) From F,= mw, ino mat mg sin O = m7 = mee mee ds RdO or, gRsin0d0= vdy Integrating both side for obtaining v (0) ° ’ J grsinoao~ fora 0 0 or, \yv? = 2.gR (1 -cos 8) From F,= mw, mg cos 0-N= m— me, At the moment the body loses contact with the surface, N= 0 and therefore the Eq. (2) becomes v= gR cos @ Q) 51 where v and @ correspond to the moment when the body loses contact with the surface. Solving Eqs. (1) and (3) we obtain cos 0 = 2 or, O= cos~* (2/3) and v= V2gR73. 188 At first draw the free body diagram of the device as, shown. The forces, acting on the sleeve are it’s weight, acting vertically downward, spring force, along the length of the spring and normal reaction by the rod, perpendicular to its length. Let F be the spring force, and Al be the elongation. From, F, = mW, Nsin 0 +F cos 0 = mor (ly where cos 0= (I+ Al. Similarly from F,= mw, Ncos@-Fsin@= 0 ot, N=Fsin 0/cos® 2) From (1) and (2) F (sin 0/cos 8) - sin 8+ F cosO= mw? r = mo (ly + Al)/cos @ On putting F= x Al, x Alsin?0+« Alcos?0= mw? (I)+A)) on solving, we get, 4 4 Al= mw? —*— = ——-__ kK-ma (k/mu*—1) and it is independent of the direction of rotation. 1.89 According to the question, the cyclist moves along the circular path and the centripetal force is provided by the frictional force. Thus from the equation F, = mw, 2 a fr= ™™ or kmg = = or b(t-g)ee% or = ky(r-17/R)g (ty 2 alr For Vaux We should have —=—* = 0 or, 1-2. 0, so r= R/2 Hence Ynys = $V Fe aR 1.90 As initial velocity is zero thus v= 2wes @ As w,>0 the speed of the car increases with time or distance. Till the moment, sliding Starts, the static friction provides the required centripetal acceleration to the car. Thus fr= mw, but frs kmg 52 191 1,92 So, Wake on eke Pe or, vs (Pg’-w)R Hence Vax = Vg = WAR we dL Th it Ymax 1 $0, from Eqn. (1), the sought distance s= >= V(t Since the car follows a curve, so the maximum velocity at which it can ride without sliding at the point of minimum radius of curvature is the sought velocity and obviously in this case the static friction between the car and the road is limiting. Hence from the equation F, = mw 60m. tong = or vs VERE so Vasax = VER in 8 + @ We know that, radius of curvature for a curve at any point (x, y) is given as, 14 (dy/dxy PA Ra | Lt @iayy 2) (yy /a? ° For the given curve, 2 2. 00s z and @- Substituting this value in (2) we get, [1 + (@7/02) cos? (x/a) P? (a/c) sin (x/a) For the minimum R, 5 and therefore, corresponding radius of curvature 2 Roig ) Hence from (1) and (2) Vinx = 0 Vga The sought tensile stress acts on each clement of the chain. Hence divide the chain into small, similar elements so that each clement may be assumed as a particle. We consider one such element of mass dm, which subtends angle do at the centre. The chain moves along a circle of known radius R with a known angular speed @ and certain forces act on it We have to find one of these forces. From Newton’s second law in projection form, F, = mw, we get 2T sin (da/2) - dN cos 0 = dm wR and from F, = mw, we get aN sind = gdm Then putting dm = mdo/2n and sin (da/2)= do./2 and solving, we get, rT. m (wR +g cot 0) 20 53 Ty fH dmg ' 1.93. Let, us consider a small clement of the thread and draw free body diagram for this element. (a) Applying Newton’s second law of motion in projection form, F, = mw, for this element, (1+ dT) sin (40/2) + Tsin (d0/2)-dN= dmw’R= 0 or, -2T'sin (d0/2)= dN, {negelecting the term(dT sin d 8/2) ] or, Td0= dN, as sin4®. 28 Q 2 2 Also, dfr= kdN= (T+dT)-T= dT t 2) From Eqs. (1) and (2), kTd0= dT or 42. ¢a0 In this case Q=x so, nek 3 oon, IngP= kx @) 1, 1 So, ke in pm into (6) When =. ny, Which is greater than My the blocks will move with same value of 1 acceleration. (say w) and clearly m, moves downward. From Newton’s second law in projection form (downward for m, and upward for m,) we get : mg-T,= mw 4 and T,-m,g= m,w ©) 54 1.94 1.95 1.96 1.97 Al h Iso 7,7 % © Simultaneous solution of Eqs. (4), (5) and (6) yields : we L~ MoM )8 _ = No) se (m+19m) (+79) m The force with which the cylinder wall acts on the particle will provide centripetal force necessary for the motion of the particle, and since there is no acceleration acting in the horizontal direction, horizontal component of the velocity will remain constant througout the motion. So Ve Vp C08 Using, F, = mw, , for the particle of mass m, ue mvt mvpcos?a NR RS which is the required normal force. Obviously the radius vector describing the position of the particle relative to the origin of coordinate is r= xityj= asinwti+bcoswtj Differentiating twice with respect the time : ao OT z os We Kye Oo (asinwti4 bcos wtj}= -0F qd) Thus F= m= -mor (a) Wehave Ap™ s Fat ' = [mziem mz @ 0 2 (v8) (b) Using the solution of problem 1.28 (b), the total time of motion, t= - = Hence using t= vin (1) 1ap"] = met = Im (weve (Fis ~~) From the equation of the given time dependence force F= @ f(t-f) at f=, the force vanishes, (a) Thus apm p= f Fat 0 55 . or, Bf aece-nyar = 0 but p= mv'so v= me -, 6m (b) Again from the equation F= mir av at(v-t)=m- or, aUtv-t?)dt= mdv” Integrating within the limits for V%t), ' f tee? ae mf av o Oo we 22 O) P(e et m 23 or, Thus 1.98 We have F = Fy sin wt dv>. Pe mG Fosinut or mdv™ Fysin ot dt or On integrating, ap. 70 ; mv = ——cos wt + C, (where C is integration constant) % When t= 0, v=0, so C= —& mo > Fo = -F Hence, = —cos wt mo mo Fy As | cos wt < 1 so, v= —*(1-cos af) mo 56 1.99 1.100 Thus s= fod Fit Fosinot Fo ( in ot) -—--——>- of - sin wt mo” mo ma” (Figure in the answer sheet). + os According to the problem, the force acting on the particle of mass m is, F = Fy cos ot v= LF So, me Focoswt or di —* cos ade dat m Integrating, within the limits. y t lo = <> Fo 3 Fog dv= cos wtdt or v= ——sin wt m mo 0 0 It is clear from equation (1), that after starting at = 0, the particle comes to rest fro the first time at t= &. F 1 > Fo. x ‘rom Eq. (1), v= |v[= ma Si OF forts o (2) Thus during the time interval ¢ = /o, the sought distance From Eq. (1) F “ms” Go | sinwe | <1 - > avo (2) From the problem = F= -rv so m9o= -1w av ~~ Thus man -w[asdvtiv] or, De fa vm On integrating Inve= ~Ftee But at t= 0, v=v, s0, C= Invy voor nh or, Ten m! Oh Y= em Thus for tro,ved av =r (©) We have m= -rv so dv= —"ds 1101 87 Integrating within the given limits ® obtain v (5): or, fon -sfe or vey (a) Thus for ve 0, 5 = Sq = 2 (©) Let we have mde ary or he at v vm ' % or, fe, of, In = Ly n% ™ 0 0 So te =m In(iM _ ming r r Now, average velocity over this time interval, ren » fucka vdt “0 Yo(n-1) g(sinax From (1) v= 0 at either cos a) @ 2 x= 0, or x= ~tana a As the motion of the bar is unidirectional it stops after going through a distance of x “vy hng una. From (1), f0F Varig di. op vi 1 & (sin ax - 7a cos a) = 0, which yields x= (tana Hence, the maximum velocity will be at the distance, x= tana/a Putting this value of x in (1) the maximum velocity, v ¥V gsinatan a ‘max @ Since, the applied force is proportional to the time and the frictional force also exists, the motion does not start just after applying the force. The body starts its motion when F equals the limiting friction. Let the motion start after time t , then F = atp=kmg ot, y= ie So, for #= sf, the body remains at rest and for £ > fy obviously me a(t=%) 0 mdv= ae-s)at Integrating, and noting v = Oat £ = tg, we have for > fy a fe-wa or v= J t-0" ‘i a Thus se fran £ fe 1.104 1.105 While going upward; from Newton’s second law in vertical direction : vdv 2 vdv a ioe ra) =) -ds ge m At the maximum height hy the speed v = 0, s0 ints - --f “ gt(k 7m) Integrating and solving, we get, m bp he Fin ( 1+ =) q) ‘When the body falls downward, the net force acting on the body in downward direction equals (mg -kv?), Hence net acceleration, in downward direction, according to second law of motion Thus Integrating and putting the value of 4 from (1), we get, v'mvo/ Vis kve/mg. Let us fix x - y co-ordinate system to the given plane, taking x-axis in the direction along which the force vector was oriented at the moment f= 0, then the fundamental equation of dynamics expressed via the projection on x and y-axes gives, dy, Feasot= m= (a) d and Fsinot= m— Q dt , oi . F. (@) Using the condition (0) = 0, we obtain v,= sine @) and y= & (1-cos.@r) @ 2a ye 2F {at Hence, ve WH (x0 va( $')| 1.106 1.107 (b) It is seen from this that the velocity v tums into zero after the time interval At, which can be found from the relation, At ot Consequentely, the sought distance, is ar sx f va ar mo ° vdt fa awe 2F_.. ot So, -f sin( $*) a/ (2n0)= 0 The acceleration of the disc along the plane is determined by the projection of the force of gravity on this plane F,= mg sino. and the friction force fr = kmg cos a. In our case k= tana and therefore Average velocity, = xmo fr= F,=mgsino. Let us find the projection of the acceleration on the derection of the tangent to the trajectory and on the x-axis : mw,= F,cos@ - fr= mg sina (cosp-1) mw, = F,- frcos p= mg sin a (1- cos @) It is seen fromthis that w, = that the velocity v and its projection v, differ only by a constant value C which does not change with time, i.e. Ww, Which means vet, where v,= vcos p. The constant C is found from the initial condition v= vo, whence 7 > initially. Finally we obtain C= vp since @ = v= vo/(1+c0s @). In the cource of time p—> 0 and v—> v9/2. (Motion then is unaccelerated.) Let us consider an element of length ds at an angle @ from the vertical diameter. As the speed of this element is zero at initial instant of time, it’s centripetal acceleration is zero, and hence, dN - Ads cos p= 0, where 2 is the linear mass density of the chain Let T and T+dT be the tension at the upper and the lower ends of ds. we have from, F,= mw, (T+ dT) +hdsgsinp-T= ddsw, or, aT+ARdggsing= dds w, 1.108 61 If we sum the above equation for all elements, the term f dT = 0 because there is no tension at the free ends, so VR rer f singdg~ dw, f dsm d1w, 0 Hence w,= SF(1-cosg] As w, =a at initial moment So, w=|w,|= 8% 100s 2 In the problem, we require the velocity of the body, realtive to the sphere, which itself moves with an acceleration Wo in horizontal direction (say towards left). Hence it is advisible to solve the problem in the frame of sphere (non-inertial frame). At an arbitary moment, when the body is at an angle @ with the vertical, we sketch the force diagram for the body and write the second law of motion in projection form F,= mw, my? ot, mg cos @ —N ~ mvp sin 0 = @) At the break off point, N= 0, 0= Q, and let v= vgso the Eq. (1) becomes, 2 Y% 1 FET 8 608 Op ~ wo sin Oy (2) From, F,= mw, vdv_ dv ds” "RdO or, vdv= R(gsin 0 +w, cos 0)d0 Q mg sin 0 - mw, cos 0 = m % 0 Integrating, fv dy =| R(gsinO + wy cosd) dO ° 0 2 3H 7 8(1 ~ £080) + Hp sin Oy @) Note that the Eq. (3) can also be obtained by the work-energy theorem A= AT (in the frame of sphere) therefore, mg (1 ~ 608 8p) + ming sin Op = 5 mvp [here mw, R sin 0, is the work done by the pseudoforce (- mw,)] 2 0 . on Fm &(1= C05 Oy) + Wo sin Oy 62 1.109 1.110 Solving Eqs. (2) and (3) we get, 24kV 549k? 3(1+k?) ™ , where k= vo = V2gR73 and | Hence 0 |.,-2 17° This is not central force problem unless the path is a circle about the said point. Rather here F, (tangential force) vanishes. Thus equation of motion becomes, Vv, Vo™ constant me A ror and, for r= ro We can consider the latter equation as the equilibrium under two forces. When the motion is perturbed, we write r= rp +x and the net force acting on the particle is, 2 2 2 m2 my my —_A_, 7% (Ee (1-f)--SR Gams " (rotx)" TotX rf m]}° % " ‘mv This is opposite to the displacement x, if n<1- (= is an outward directed centrifugul force while =A is thé inward directed external force). r There are two forces on the sleeve, the weight F, and the centrifugal force F,, We resolve both forces into tangential and normal component then the net downward tangential force on the sleeve is, 2 mg sin6 {1 - “Res e This vanishes for 0= 0 and for 0 @= 0, = cos” (—8—|, which is real if oR wR MwW?RSin10CosO wo R>g. If oR g, 1- cos0 is negative for small © near 6 = 0 and 6 = 0 is then unstable. However 0 = 0g is stable because the force tends to bring the sleeve near the equilibrium position 6 = Oy. If wR = g, the two positions coincide and becomes a stable equilibrium point. 1 1112 1.113 63 Define the axes as shown with z along the local vertical, x due east and y due north. (We assume we are in the northern hemisphere). Then the Coriolis force has the components. Fo = -2m@xv) = 2m | v, cos8 - v, sind) i= v, cos® j + v, cos® k} = 2ma (v, cosO - v, sind) i” since v, is small when the direction in which the gun is fired is due north. Thus the equation of motion (neglecting centrifugal forces) are = mo (v, sing - v,cosq), j= 0 and z= -g Integrating we get y= v (constant), Z= - gt and X= 2w vsing t+ wg t? cosp Finally, xe ov? sing + 3 gt? cose Now v >> gt in the present case. so, 2 2 x= ovsing (=) = wsing = v v ~ 7m (to the east). The disc exerts three forces which are mutually perpendicular. They are the reaction of the weight, mg, vertically upward, the Coriolis force 2mv’ w perpendicular to the plane of the vertical and along the diameter, and mw’r outward along the diameter. The resultant force is, FamVe+0'r+(Qv oy The sleeve is free to slide along the rod AB. Thus only the centrifugal force acts on it. The equation is, . og dr mia mo?r where v= 2. de Buti= ye #(3") dr dr\2 0, 1. 1027+ constant 2 2 or, Po hea? ¥p being the initial velocity when r= 0. The Coriolis force is then, Ama Vv2 + oF? = 2mw*r V1 + 2/0 PF = 2.83 on putting the values. 64 1.114 1.115 The disc OBAC is rotating with angular velocity w about the axis OO’ passing through the edge point O. The equation of motion in rotating frame is, mv = F's mu? R's Imvxd= F+F, where F,, is the resultant inertial force (pseudo force) which is the vector sum of centrifugal and Coriolis forces. (a) AtA, F,, vanishes. Thus 0= -2mw?R n+ 2mv' on a Cc where 7 is the inward drawn unit vector to the centre from the point in question, here A. Thus, v= oR 0, wee (b) ALB F,= mo? OC + mo? BC its magnitude is ma” VaR" - r*, where r= OB. The equation of motion in the rotating coordinate system is, mW" = F's mo*R+ 2m (vx B) Now,’ v= RO@%+Rsind per and W = wcos 02, -w’ sin 0 2 ~ oS > 11> e 9 eo Sy Few™ 0 RO Rsindo @cos 8 -wsin® 0 = 2 (oR sin’0 @) + OR sin 0 cos 6 p 24 - WR 0 cos 6 2p Now on the sphere, v= (-RO-Rsin? 0g") a + (RO -Rsin@ cos 0g”) & + (Rsin Og" + 2R cos 009) Thus the equation of motion are, m(-R@-R sin? 6G”) = N- mg cos 0 + mu? R sin? @ + 2mw R sin? 6 @ m(R@ -Rsin 8 cos 6g”) = mg sin 0 + mw” R sin 6 cos 6 + 2mw R sin 8 cos 8p m(Rsin 0 @ +2R cos 60) = —-2 mw R 6 cos 0 From the third equation, we get, P= -@ A result that is easy to understant by considering the motion in non-rotating frame. The eliminating @ we get, | mR @ = mg cos @-N mRO = mgsin® Integrating the last equation, J mR = mg (1 - cos 6) 1116 Lu7 65 Hence N= (3-2 cos 8) mg So the body must fly off for = 6,= cos~*2, exactly as if the sphere were nonrotating. Now, at this point F,y= centrifugal force = mo? R sin 0) = vi mo? R Fug = Vo R’ 0 cos" 0 + (wR) sin’ © x 2m Vet 2 py 4, 28 = ma? Rv 8s 5 RY +0? R’ XOX 3p X2m= FmorR 5+ aR (a) When the train is moving along a meridian only the Coriolis force has a lateral component and its magnitude (see the previous problem) is, 2m wv cos ®= 2m sind (Here we have put R 6-> v) So, Fauna 2% 2000 x 10? x 2, $4000, VE 36400 * 3600 “2 = 3-77KN, (Wwe write for the latitude) (b) The resultant of the inertial forces acting on the train is, ao SSN Feor y F,, = ~ 2mw R 6 00s 02 + (ma? R sin 0 cos 0 + 2m wR sin 8 cos 8) & + (ma? R sin? 6 + 2m w R sin’ 0G) er This vanishes if = 0, @= -F0 1 . 1 Thus We Vg Bs Vem FOR sin Om - aR cos (We write % for the latitude here) Thus the train must move from the cast to west along the 60" parallel with a speed, 2n x 1074 x 6-37 x 10° = 115-8 m/s ~ 417 km/hr daRcosr= 4x 2% We go to the equation given in 1.111. Here v, = 0 so we can take y= 0, thus we get for the motion in the x2 plane, ¥ = ~20v,c08 8 and Zang Integrating, ze ge = wgcos gt? So x= do pcos ptt= 1 agcosp 3 3 = 28h seg VE 3 PVE There is thus a displacement to the east of 2,28 af 22 5% g 64x 500x1x VY 55 ~ 26cm. 66 1.3 1.118 1119 1.120 1121 Laws of Conservation of Energy, Momentum and Angular Momentum. As Fis constant so the sought work done A= Fare F-G3-72) tk ce om meee ee o, A= (31447) [I-37 )- G42) = (31447) (ie 5] = 17I Differentating v (s) with rere to time #. a a - ie e aetna won locomotive is in unidrectional 1 moto) Hence force acting on the locomotive F =m w= me Let, at v= Oat ¢ = 0 then the distance covered during the first t seconds Hence the sought work, A= Fs= "2 @72 2 4 8 We have 1 2 2as? Ts zm = as or, v= 7 a Differentating Eq. (1) with respect to time 4as 2as 2vm,= Sy on we (2) Hence net acceleration of the particle 7 we Vw + 208 WTS RY m mR m Hence the sought force, F = mw= 2as V1 + (s/RY Let F makes an angle 6 with the horizontal at any instant of time (Fig.). Newton’s second Jaw in projection form along the direction of the force, gives : F = kang cos 0 + mg sin @ (because there is no acceleration of the body.) As F tt d7the differential work done by the force F, Pe > & dA= F-d7™ Fds, (where ds = | d7"|) N = kang ds (cos @) + mg ds sin oP) h = king dx + mg dy. 1 h x fr Hence, A = kang | dx+mg | dy 0 0 & = king 1+ mgh= mg (k1+h). x 67 1122 Let s be the distance covered by the disc along the incline, from the Eq. of increment of ME. of the disc in the field of gravity : AT+ AU™ Ay, 0 + (~ mgs sin a) = — king cos as - kmg I or, @) Hence the sought work Ay, = ~ kmg [s cos a +1 if 8 [ ] 2g -- kim: ‘ Ap Toe poke [Using the Eqn. (1)] <—] On puting the values A, = -0.05 J mg 1123 Let x be the compression in the spring when the bar m, is about to shift. Therefore at this moment spring force on m, is equal to the limiting friction between the bar m, and horizontal floor. Hence «x= km,g [where x is the spring constant (say)] Q) For the block m, from work-energy theorem : A= AT = 0 for minimum force. (A here indudes the work done in stretching the spring.) 50, Fr-}xx?—kmgr= 0 or Kp F-kng Q, From (1) and (2), m2 1 tne} 1424 From the initial condition of the problem the limiting fricition between the chain lying on the horizontal table equals the weight of the over hanging part of the chain, i.e. Anig= kA(1—n) Ig (where 2 is the linear N mass density of the chain) So, ke 1-7 OF Let (at an arbitrary moment of time) the length ‘of the chain on the table is x. So the net friction force between the chain and the table, at this Ax moment : 3 Adx) f= N= kag Q) d The differential work done by the friction forces : GA = fy-d7= -f,ds-= —krxg(-de)= dg Th xde @) (Note that here we have written ds = ~ de, because ds is essentially a positive term and as the length of the chain decreases with time, di is negative) Hence, the sought work done 0 Aw f reqhoeaee -a- -w ntl. -135 Q-n)t 1.125 1.126 1.127 The velocity of the body, ¢ seconds after the begining of the motion becomes V= vo +g. Tht power developed by the gravity (m g’) at that moment, is P= mg” v= m(B”- 19+ 8't) = mg (gt- vp sina) (1) As mg” is a constant force, so the average power P= A. mie Ar” T T where AF” is the net displacement of the body during time of flight. As, mg LAr” so

= 0 2 We have wee gra, or, v= VaRt, 1 is defined to start from the begining of motion from rest. av So, Me ae . . Instantaneous power, P= F += m(w,u,+, u, )-(VaR tu, ), (where i, and i, are unit vectors along the direction of tangent (velocity) and normal respectively) So, P= mw,VaR t= ma Rt Hence the sought average power * 2 ma Rt 2 Let the body m acquire the horizontal velocity vp along positive x — axis at the point O. (a) Velocity of the body ¢ seconds after the begining of the motion, Ve + Wee (vy—kgt)i” Q) Instantaneous power P= Fi" (- kmgi')+(vy- ket) im ~ kang (vp — ket) From Eq. (1), the time of motion t= vo/kg Hence sought average power during the time of motion * Hence - 2 'W (On substitution) a, = kang = mw, my, or, v,dvg= —kgde = - agxde 1.128 1.129 1130 To find v (x), let us integrate the above equation ’ = f nya ag f xde or, Ve -age (a) 0 " Now, Ba Fv -maxgVii-age Q) For maximum power, fea = 0 which yields x Putting this value of x, in Eq. (2) we get, Pause bm iBVaE Centrifugal force of inertia is directed outward along radial line, thus the sought work ey A= f mo” r dr = 5 mu? (3 - ) = 0-20T (On substitution) mn Since the springs are connected in series, the combination may be treated as a single spring of spring constant. KK k= Ky +k, From the equation of increment of ME, AT+AU=A,, Orin Al?= A, of A= 5 bc] Jar 2 2| Ky +K, First, let us find the total height of ascent. At the beginning and the end of the path of velocity of the body is equal to zero, and therefore the increment of the kinetic energy of the body is also equal to zero. On the other hand, in according with work-energy theorem AT is equal to the algebraic sum of the works A performed by all the forces, ie. by the force F and gravity, over this path. However, since AT= 0 then A = 0. Taking into account that the upward direction is assumed to coincide with the positive direction of the y-axis, we can write h hs A= f Fenih-a *Se,-mow 0 s - mg f (1-209) dy= mgh (1 - ah) = 0. whence h = 1/a. The work performed by the force F over the first half of the ascent is mr nr Ape f Fay amg faa) ay 3 mg/4a, 0 0 The corresponding increment of the potential energy is AU= mgh/2= mg/2a. 70 1.131 1,132 From the equation F, = - 2 we get F,= | -2% +5 ar r r (a) we have at r= ro, the particle is in equilibrium position. ie. F,= 0 80, ro= 0 To check, whether the position is steady (the position of stable equilibrium), we have to satisfy 2 <2 >0 @U_[6a_ 2 we nave 2. ($2) Putting the value of r= ry= 2, we get 2 4 ot. = (as a and b are positive constant) 2 So, du, # >0, dr?” 8a which indicates that the potential energy of the system is minimum, hence this position is steady. (b) We have dF, For F, to be maximum, "= 0 3a -e So, rm $F and then F, (ay) ™ Se As F, is negative, the force is attractive. (a) We have au -aU F,= -9o" -2ax and Fy= a -2By F. Jeopyis Veda BP So, Fe 2axi-2Byiand, Fe 2Vo7r +p y a For a central force, 7x F= 0 Here, 7xFe (xityj )x(-2axi-2Byj) = -2Bxyk-2axy(e)«0 Hence the force is not a central force. () As U= ax? +By? aU -3aU So, Fw Spr 72ax and B= Som -2By. So, FeVFsF =Vaarxr+4py According to the problem F= 2Vo0'x+ fy = C (constant) 133 114 1135 71 a ovine 2 ¢ or, ole Dart” Fav) Q) Therefore the surfaces for which F is constant is an ellipse. For an equipotential surface U is constant. So, ax?+By*= Co (constant) or, x wt. Lo, K, (constant) VE Ve” a8 Hence the equipotential surface is also an cllipse. Let us calculate the work performed by the forces of each field over the path from a certain point 1 (x,, y,) to another certain point 2 (x), y,) 5 @ dA= F-dr= ayitd7™ ayde or, A= a fra > >. => A (ii) dA= F-d7™ (axi+byi)-d7™ axdx + bydy %y Ma Hence Ao f axdee bydy AL n In the first case, the integral depends on the function of type y (x), ie. on the shape of the path. Consequently, the first field of force is not potential. In the second case, both the integrals do not depend on the shape of the path. They are defined only by the coordinate of the initial and final points of the path, therefore the second field of force is potential. Let 5 be the sought distance, then from the equation of increment of M.E. AT +AU = Ay, ( ~pImd) + (+mes sina = —kng cosas 2 %o ; = 2/(ina+k or, s 3, / (in cos «) ~ kena Hence Ap = ~kong cos = cay Velocity of the body at height h, v, = V2g(H—/), horizontally (from the figure given in the problem). Time taken in falling through the distance 2. t= V (as inital vertical component of the velocity is zero.) Now s=vt= Vigith VF - Vac 72 1.136 1.137 For Snap 4 (Hh ~ h?) =0, which yields h -# Putting this value of A in the expression obtained for s, we get, Seux = H To complete a smooth vertical track of radius R, the minimum height at which a particle starts, must be equal to Sr (one can proved it from energy conservation). Thus in our problem body could not reach the upper most point of the vertical track of radius R/2. Let the particle A leave the track at some point O with speed v (Fig.). Now from energy conservation for the body A in the field of gravity : hae 1 me[n-30 sino] am or, v= gh(1-sin®) (a) From Newton’s second law for the particle at the point O; F,= mw,, . mv? N+mgsin0= wD 1H But, at the point O the normal reaction N= 0 So, ve Bh sn 8 (2) From (3) and (4), sin 0 = 2 and v= V7 5 After leaving the track at O, the particle A comes in air and further goes up and at maximum, height of it’s trajectory in air, it’s velocity (say v’) becomes horizontal (Fig,). Hence, the sought velocity of A at this point. A ‘i 24/gh v= vcos (90-8) = vsin@= eV 5 Let, the point of suspension be shifted with velocity v, in the horizontal direction towards left then in the rest frame of point of suspension the ball starts with same velocity horizontally towards right. Let us work in this, frame. From Newton’s second law in projection form towards the point of suspension at the upper most point (say B) : 2 2 mv) my; mg+T= 7" 01, T= —7-mg (a) Condition required, to complete the vertical circle is that T2 0. But 2) Sm = mg (2) +5 mie So, Y= 2-4g1 @) 1.138 1139 73 From (1), (2) and (3) (v3 - 4 gl T- mA ase) geo or, vx VSql Thus M4 (gig = V5 gl From the equation F, = mw, at point C me rea @ Again from energy conservation dm = 5 mv? + mel Va © From (4) and (5) T= 3mg Since the tension is always perpendicular to the velocity vector, the work done by the tension force will be zero. Hence, according to the work energy theorem, the kinetic energy or velocity of the disc will remain constant during it’s motion. Hence, the sought time t= a where s is the total distance traversed by the small disc during it’s motion. 5 Now, at an arbitary position (Fig.) ds= (Iy-R0)d0, UR so, s=f @-Roa0 0 Ree or, SR a” wR Hence, the required time, t= IRy It should be clearly understood that the only uncompensated force acting on the disc A in this case is the tension T, of the thread. It is easy to see that there is no point here, relative to which the moment of force T is invarible in the process of motion. Hence conservation of angular momentum is not applicable here. Suppose that AJ is the elongation of the rubbler cord. Then from energy conservation, AU, +AU, = 0(as AT = 0) or, -mg (+A) +hn Al?» 0 on, Sn Al? mg Al—mgi = 0 74 1.140 1.141 or, Al= a/ « mgs V (my +4xFmgl ne [ za | — ye 2 KL me axl a7 Lt 1th 2x % Since the value of V1 + x is certainly greater than 1, hence negative sign is avoided. ~me(,/ za) So, Al mes ei When the thread PA is burnt, obviously the speed of the bars will be equal at any instant of time until it breaks off. Let v be the speed of each block and © be the angle, which the elongated spring makes with the vertical at the moment, when the bar A breaks off the plane. At this stage the elongation in the spring. Al= [sec 0 ~ Ip = Ip (sec 0 ~ 1) @ Since the problem is concerned with position and there are no forces other than conservative forces, the mechanical energy of the system (both bars + spring) in the field of gravity is conserved, ic. AT+AU = 0 So, 2 (jm +5 19 Gee -1)?—mslytanO= 0 2) From Newton’s second law in projection form along vertical direction : mg = N+ ly (sec 0-1) cos 8 Klo Gac6-1 But, at the moment of break off, N= 0. Hence, x Jp (sec @ - 1) cos @ = mg Kh or, cos 0 = 8 @) Kh Taking x = 58, simultaneous solutionof (2) and (3) yields : h 19g veV 32 Obviously the elongation in the cord, Al= / (sec @~ 1), at the moment the sliding first starts and at the moment horizontal projection of spring force equals the limiting friction. So, x, Alsin = kN @) 17m/s. (where x, is the elastic constant). KAL From Newton’s law in projection form along vertical direction : x, Alcos @+N= mg. or, N= mg-x, Alcos From (1) and (2), K, Alsin @ = k (mg - x, Alcos 6) mg 1.142 1.143 7 a Alsin 8 + k Al cos 8 From the equation of the increment of mechanical energy : AU + AT = Ay, 1 or, (gma) = Ae king Al? ZAl (sin 6 + k cos 0) Aang I, (sec 0 - 1) 2 (sin 0 — Koos 0) Let the deformation in the spring be Al, when the rod AB has attained the angular velocity . From the second law of motion in projection form F, = mw, . oO, Ky or, = Ay Thus Ay, = = 0-093 (on substitution) 2 mor k Al= mo? (Iy+A) or, Al= ° KAl= ma’ (+A on x- mor From the energy equation, A, = dmv? +x al? Lia? 2d Ape = zi (lp+ Al? +5 x Al a wnat (y+ mY 4 Lg (me 2 (0+ Tima? | *2™ Cem? xanit+n) mo On solvinj Ag £2, where qe 7 * "2 aw ms We know that acceleration of centre of mass of the system is given by the expression. im, 4m Ree Since Wx -W, —» _ (m,-m,) Wy Wem mn @ m, +m, Now from Newton’s second law F= mi, for the bodies m, and m, respectively. 7 = T+ m, B= mW, Q and Fe m,g= mw,=-mw, (3) Solving (2) and (3)

1 > > T= xm (B-V P+ zm, (Z-V) This is minimum with respect to variation in V, when road 0, ie. m, (9, - VY? +m, (9,-V) =0 Ve > > => mv, +m, V, or ea m, +m, Hence, it is the frame of C.M. in which kinetic energy of a system is minimum. (6) Linear momentum of the particle 1 in the K’ or C frame MyM, > oo Phe my (HH = 9) >. my My or, P= w(7,-¥)), where, p= mem reduced mass Similarly, Pos w (73-7) So, IPrl= IPEl= B= Wryq where, vq= 1¥7-¥5] @) Now the total kinetic energy of the system in the C frame is 2 a 7-7F47-L,2.2 T-T,+T,- S74 oa” Op 2 x1 Lio Hence To Javan gh | MH] 78 1.148 1.149 1.150 To find the relationship between the values of the mechanical energy of a system in the K and C reference frames, let us begin with the kinetic energy T of the system. The velocity of the i-th particle in the K frame may be represented as ¥7= ¥;+ V2. Now we can write 1 ar er Ta bmvt= Sb 400) =D sR Ey mite yD Emee Since in the C frame }) m,¥;= 0, the previous expression takes the form T= Trim we F+5mv? (ince according to the problem vo= V) a) Since the internal potential energy U of a system depends only on its configuration, the magnitude U is the same in all refrence frames. Adding U to the left and right hand sides of Eq. (1), we obtain the sought relationship E- Esdm v2 As initially U= O= 0, so, E= T From the solution of 1.147 (b) a Fd ult-w, As wl Fe 1mm ‘Thus I mem TD) Velocity of masses m, and m, after f seconds are respectively. Wf = H+ gh and Hf’ = +R Hence the final momentum of the system, B= my 0 +m, Vy = m, Vf + mV; + (Mm, +m) BT = Bee mart, (where, Fo= mh +m, i and m= m,+m,) And radius vector, re Heb Wee? m,Vy+m,Va)t Aw (my vy + mz V2) sige (m, +m) 2 <1 <2 MV +m, Vy = mtekgt?, where y= MUAY 2 m, +m, 79 1.151 After releasing the bar 2 acquires the velocity v,, obtained by the energy, conservation : 1.152 1.153 dmg vg= Fux? on y= fF a Thus the sought velocity of CM. O+mx Ving _xVige Yom" - eve mm, +m” (m,+m,) Let us consider both blocks and spring as the physical system. The centre of mass of the system moves with acceleration a = ———— towards right. Let us work in the frame of 1 centre of mass. As this frame is a non-inertial frame (accelerated with respect to the ground) we have to apply a pseudo force m,a towards left on the block m,and m, a towards left on the block m, As the center of mass is at rest in this frame, the blocks move in opposite directions and come to instantaneous rest at some instant. The elongation of the spring will be maximum or minimum at this instant. Assume that the block m, is displaced by the distance x, and the block m, through a distance x, from the initial positions. From the energy equation in the frame of C.M. AT+U-A,,, (where A, also includes the work done by the pseudo forces) Here, AT=0, Unde txy? and F-mF m,F m,F (x, +3) m,+m, ttm m, +m), 1 2 my +2) F or, zk ly tm) = mam so ° 2m,F , Xt = 0 on x + x, = p+ 1 tXy 1 +2" Fon, +m) 2m,F Hence the maximum separation between the blocks equals : [y + ————— k(m,+m,) Obviously the minimum sepation corresponds to zero elongation and is equal to ly (@) The initial compression in the spring Al must be such that after burning of the thread, the upper cube rises to a height that produces a tension in the spring that is atleast equal to the weight of the lower cube. Actually, the spring will first go from its compressed 80 1.154 state to its natural length and then get elongated beyond this natural length, Let / be the maximum elongation produced under these circumstances. Then klemg @ Now, from energy conservation, Pu Al = mg (Med eda? @ (Because at maximum elongation of the spring, the speed of upper cube becomes zero) From (1) and (2), 2g ay? 2mg Al 3 = 0 oF, Al= 228, mm « Therefore, acceptable solution of Al equals ome (b) Let v the velocity of upper cube at the pos: breaks off the floor, then from energy conservation. (Gay, at C ) when the lower block 1 1 292 qm = 7K (Al ~1?)— mg (i+ Al) (where I= mg/x and Al = 778) or, v= 32 Q) K mv+0 2m At the position C, the velocity of C.M; vo= = F Let the CM. of the system (spring+ two cubes) further rises up to A yey Now, from energy conservation, 5 Om) B= Qm)g brea 2 Yo Ame or, yea" 3g" ag” ° But, uptil position C, the CM. of the system L has already elevated by, Aye = Qithms0_ dmg 2m K Hence, the net displacement of the C.M. of the system, in upward direction Aye= Aye + Ayeg= SE . Due to ejection of mass from a moving system (which moves due to inertia) in a direction perpendicular to it, the velocity of moving system does not change. The momentum change being adjusted by the forces on the rails. Hence in our problem velocities of buggies change only due to the entrance of the man coming from the other buggy. From the 1.155 1.156 1.157 81 Solving (1) and (2), we get v= and v= 1" M-m 2" M-m As Wty Vand vette” —» <= = MV So, v= and yy= 1" M-m) 2" (M-m) From momentum conservation, for the system “rear buggy with man” (M+ m)vo= m (Us vg) + Mig @ From momentum conservation, for the system (front buggy + man coming from rear buggy) Mig+ m(it vy) = (M+m) vp ge Em ?" M+m" M+m Putting the value of ¥g from (1), we get So, aR) (i Let vf be the velocity of the buggy after both man jump off simultaneously. For the closed system (two men + buggy), from the conservation of linear momentum, Mij+2m@+vz)= 0 =o 2m” or, "1" Ma om @ (ii) Let 9” be the velocity of buggy with man, when one man jump off the buggy. For the closed system (buggy with one man + other man) from the conservation of linear momentum : O= (M+m)v" +m(u40") (2) Let v3 be the sought velocity of the buggy when the second man jump off the buggy; then from conservation of linear momentum of the system (buggy + one man) : (M+ m)¥" = Miz +mU+¥;) @) Solving equations (2) and (3) we get or, (2M +3m)i” @ "2° (a+ m) (M+ 2m) From (1) and (4) v2 m 47 1 *2@tem)?* Hence v, > ¥; The descending part of the chain is in free fall, it has speed v= V2 gh at the instant, all its points have descended a distance y. The length of the chain which lands on the floor during the differential time interval dt following this instant is vdt. 1.158 1.159 For the incoming chain element on the floor : From dp, = F, dt (where y-axis is directed down) ! yy 0- (avd) v= F, de or Fy= -Avm -2dgy Hence, the force exerted on the falling chain equals A and is directed upward. Therefore from third law the force exerted by the falling 1 chain on the table at the same instant of 4-8 => time becomes Av? and is directed downward. ¢ Mv Since a length of chain of weight (Ayg) already lies on the table the total force on the floor is (2Ayg) + (Ayg) = (3Ayg) or the weight of a length 3y of chain. Velocity of the ball, with which it hits the slab, v= V2 gh After first impact, v= ev (upward) but according to the problem v’ = . soe= x @ and momentum, imparted to the slab, = mv-(— mv’) = mv(1 +e) Similarly, velocity of the ball after second impact, Wiseve ey And momentum imparted = m (v4 v")= m(1+e)ev Again, momentum imparted during third impact, = m(1+e)ev, and so on, Hence, net momentum, imparted = mv (1 +e) + mve (1 +e) +mve*(1+e) +... = mv(L+e)(lte+e'+...) my fea, (from summation of G.P.) ( +t 7 = V2. gh 75 = mV2 gh / (n+ 1)/ (n-1) (Using Eq. 1) y-4 (4) = 0:2kg m/s. (On substitution) (a) Since the resistance of water is negligibly small, the resultant of all external forces acting on the system “a man and a raft” is equal to zero. This means that the position of the CM. of the given system does not change in the process of motion. ie. 7E= constant or, ArZ= 0 icc. S) m,A7/= 0 or, m (BFnae + Aig) +M Aig, =0 Thus, m(I”' +1)+MI= 0, or, T=- ml ° * m+M (b) As net external force on “man-raft” system is equal to zero, therefore the momentum of this system does not change, So, O= ml" +H O]+MIZO 1.159 1.160 1.161 1.162 83 (@) Since the resistance of water is negligibly small, the resultant of all external forces acting on the system “a man and a raft” is equal to aero. This means that the position of the C.M. of the given system does not change in the process of motion. ie. 7E= constant or, Arc¢= 0 ie. Sm, Ar;= 0 or, m (Arman + Arg) + M Ari, -0 _ . ml” Thus, m(T”'+1)+MI= 0, or, 1-7 (b) As net external force on “man-raft” system is equal to zero, therefore the momentum of this system does not change, So, O= m[o" (N+ 14M) on, Ho- -ma @) As ¥” (1) or ¥3(¢) is along horizontal direction, thus the sought force on the raft Maz Mm av") dt m+M = dt Note : we may get the result of part (a), if we integrate Eq. (1) over the time of motion of man or raft. In the refrence frame fixed to the pulley axis the location of C.M. of the given system is described by the radius vector we M Aig + (M-m) ATag_ mp tt Wn Aree OM But Afy= ~AFay—m and ATE AF rap m + AToe—m mI” Th Ar= 2 us TC" OM Note : one may also solve this problem using momentum conservation. Velocity of cannon as well as that of shell equals V2 g/sin a down the inclined plane taken as the positive x-axis. From the linear impulse momentum theorem in projection form along x - axis for the system (connon + shell) ic. Ap, = F, At: poosa-MV2gisina = Mgsina At (as mass of the shell is neligible) posa-MV2gisina or, Are Boson veges Mg sina From conservation of momentum, for the system (bullet + body) along the initial direction of bullet mv m+M mvy= (m+M)v, of, v= 84 1.163 When the disc breaks off the body M, its velocity towards right (along x-axis) equals the 1.164 velocity of the body M, and let the disc’s velocity’in upward direction (along y-axis) at that moment be v', From conservation of momentum, along x-axis for the system (disc + body) my 1 m+M a And from energy conservation, for the same system in the field of gravity : Sm? = F0m+Mv2+dmv2 +mgh', mv = (m+M) vi, oF v= where h’ is the height of break off point from initial level. So, 1 mv 1 min 5 2+) emt 2" 2 + mh’, using (1) or, Ve tp ag Also, if h” is the height of the disc, from the break-off point, then, vim 2gh" Ss 2g (h +h')= amy °» 8 7 (+m) Hence, the total height, raised from the initial level = ia hte Me 2g (M+ m) (@) When the dise slides and comes to a plank, it has a velocity equal to v= V2 gh. Due to friction between the disc and the plank the disc slows down and after some time the disc moves in one piece with the plank with velocity v’ (say). From the momentum conservation for the system (disc + plank) along horizontal towards right : my m+M Now from the equation of the increment of total mechanical energy of a system : my = (m+M)v' or v= 1 al 5M +m) v?— > my» Ay, 1 vol or, 7M) me ZI Ase 1 2[_ So 2” [ata |= 4 mM Hence, Ant - (ren) = ugh (were be courte reduced mas) 1.165 1.166 (©) We look at the problem from a frame in which the hill is moving (together with the disc on it) to the right with speed u. Then in this frame the speed of the disc when it just gets onto the plank is, by the law of addition of velocities, v = u +V2gh. Similarly the common speed of the plank and the disc when they move together is ~ m Von us VR. Then as above 4, = Lom+My¥? ~ Limi? - Lan? - d(m+M) {esta BR at - Sms Mu? 4m 2u VIR - meh We see that Ay, is independent of u and is in fact just - 4 gh as in (a). Thus the result obtained does not depend on the choice of reference frame. Do note however that it will be in correct to apply “conservation of enegy” formula in the frame in which the hill is moving. The energy carried by the hill is not negligible in this frame. See also the next problem. In a frame moving relative to the earth, one has to include the kinetic energy of the earth as well as carth’s acceleration to be able to apply conservation of energy to the problem. In a reference frame falling to the earth with velocity v,, the stone is initially going up with velocity v, and so is the earth. The final velocity of the stone is 0 = v,-gt and that of the earth is v, + 77 8¢ (M is the mass of the earth), from Newton’s third law, where ¢ = time of fall. From conservation of energy 2 Aime + Sand + mgh = 7M (r + i) Hence 5% (- + i] = mgh Negecting a in comparison with 1, we get Vj = 2gh or v, = V2gh The point is this in earth’s rest frame the effect of earth’s accleration is of order 5 and can be neglected but in a frame moving with respect to the earth the effect of earth’s acceleration must be kept because it is of order one (i.e. large). From conservation of momentum, for the closed system “both colliding particles” myvy + mai, = (mm, +m)" my, + my in 27) j-6k) >.> > pe mbt my 1Gi-2j)+2j-6b) 7 97 gz my +My 3 Hence |v] = V¥14+4+16 m/s= 46m/s or, 1.167 For perfectly inelastic collision, in the C.M. frame, final kinetic energy of the colliding 1.168 system (both spheres) becomes zero. Hence initial kinetic energy of the system in CM. frame completely turns into the internal energy (Q) of the formed body. Hence 7 1 2 Q= Fm hula-ay Now from energy conservation AT = -Q = - In lab frame the same result is obtained as wl (mitt mr 1 ei em iar ar = 5 ee 5m, Lie? +m, IL =~ 5ula- a? (a) Let the initial and final velocities of m, and m, are ij, i and V7 V9 respectively. Then from conservation of momentum along horizontal and vertical directions, we get : m, uy = mv, cos 0 @) and my, = m3v,sin Q ty Squaring (1) and (2) and then adding them, 12,2. 24,2 2 u mivz= mi (uh +4) -O> Now, from kinetic energy conservation, J N 1 1 1 . TMM Tmt SM @) Va mi or, m(u-v)= myz= m3 (+ Vi) [Using (3)] 2 m, m, or, wWi1-— = Ble my, my Vy ? m, on (2) mm 4 wy m+m, So, fraction of kinetic energy lost by the particle 1, Lin Lim 2 gman 7m aH 222 4 ind my-m, 2m, 71am” mem, Using A) ©) (b) When the collision occurs head on, muy = MV, + MyV2 () and from conservation of kinetic energy, 1.169 87 2e1 yal zm onan °s uy (Emm) 6 Fraction of kinetic energy, lost wi ‘my= my)" Amy my =1--}-1- -—-, 6 z = = ral ease Using (8) (2) When the particles fly apart in opposite direction with equal velocities (say v), then from conservatin of momentum, m,u+0= (m,-m,)v @) and from conservation of kinetic energy, imum dm Pad mF 2 or, mw = (m,+m,) 7 Q) From &q, (1) and (2), mw mw = (mm) ae mi ---CP> ween eee or, m,-3m,m,= 0 m, Hence “= 5 asm, #0 (>) When they fly apart symmetrically relative to the initial motion direction with the angle of divergence @ = 60°, From conservation of momentum, along horizontal and vertical direction, my uy = m, v, cos (8/2) + my v, cos (8/2) Q) and my v, sin (8/2) = m, v, sin (8/2) or, m,¥, = mz; Q) Now, from conservation of kinetic energy, 1 1 1 zm w+ O= 5m, A+ 5m vi ©) From (1) and (2), my Vy m, uy = cos (8/2) (» y+ )- 2m, v, cos (8/2) 1.170 So, 4, = 2v, cos (0/2) (4) From (2), (3), and (4) 23 4 m, cos? (0/2) vem my v3 4 27 mm or, 400s? (0/2) = 144 my nm o, he dens? 8-1 m, 2 m and putting the value of 0, we get, ™ =2 If (v4.,%yy) are the instantaneous velocity components of the incident ball and (Yq Mzy)) ate the velocity components of the struck ball at the same moment, then since there are no external impulsive forces (i.e. other than the mutual interaction of the balls) We have Usina= vy» v= MU COSC. =m Vig + I Vag The impulsive force of mutual interaction satisfies da F da Zo) E-- Lon) (F is along the x axis as the balls are smooth. Thus Y component of momentum is not transferred.) Since loss of K.E. is stored as deformation energy D, we have e_imu2-nv2— Len? Da 5mut— Smvy?— Sav 1 2costa — Lmy, 2 Loy. 2 zm? cosa — sev? — Savoy - ral moose ~ my? (mucosa mvy,)? | -+ [ 2mPucosari, mi? ] = m(v,soosa~ v4.) 2 wn | ease ( uso, 4 2 be ‘We see that D is maximum when u cos and Drax Prax 12 1 a Then = Tm F008 F 2 On substiuting a= 45° x 1171 1.172 89 From the conservation of linear momentum of the shell just before and after its fragmentation wees @ where V7, ¥7 and ¥3 are the velocities of its fragments. From the energy conservation 3nv?=vi+v34+v5 Q) Now Hor if = 97-0 97-77 ® where V2 = "= velocity of the C.M. of the fragments the velocity of the shell. Obviously in the CM. frame the linear momentum of a system is equal to zero, so V+ +5=0 4) Using (3) and (4) in (2), we get By? = VEY + OEP + OL HO? = v2 + 292 + 203 4207-0 or, 25? + 25, 5, cosO +273 +3 (1-n)?* =0 () If we have had used ¥>=- v1 - V5, then Eq. 5 were contain 7 instead of ¥, and so on. The problem being symmetrical we can look for the maximum of any one. Obviously it will be the same for each. For 9,0 be real in Eq. (5) 493 cos’@ = 8(273 +3 (1 - n) ) or 6(n - 1)’ 2 (4 - cosy} So, hsv sa-) ot Tym V2(=1) v 4 = cos’ Hence vo (aa [7% Vlg =¥+V2 (1) vev(1sV2m= =1 km/s Thus owing to the symmetry Yygmax)™ V2 (na) ™ Vana) ¥ (1+V2(q)- 1) = 1 km/s Since, the collision is head on, the particle 1 will continue moving along the same line as before the collision, but there will be a change in the magnitude of it’s velocity vector. Let it starts moving with velocity v, and particle 2 with v, after collision, then from the conservation of momentum mu = mv, +mv, of, u= V4 +V2 q@) And from the condition, given, poem bt 2 1+ “TI ym or, Vieg= -ne Q) From (1) and (2), vi+(u-v) = (L-n) or, view -2u,+v= (l-n)w? 1.173 1.174 or, w-wu+nu= 0 Vagcaa So, y= du 2 eS = }[ueViP- nw? l- duQevi=m ) Positive sign gives the velocity of the 2nd particle which lies ahead. The negative sign is correct for v, . So, 4, = du (1-VI=2H ) = 5m/s will continue moving in the same direction. Note that v,= 0 if = 0 as it must. Since, no external impulsive force is effective on the system “M + m”, its total momentum along any direction will remain conserved. So from p, = const. u mum My, cos® of, v= 5G a) and from p, = const my,= Mv,sin® ot, v,= “y,sinO= utan@, [using (1)] Final kinetic energy of the system T= mi + img And initial kinetic energy of the system= Smut 1, -T, So, % change = a x 100 i 1 1 voli < mu tan? 045 ue 2 2" costo 2” - i x 100 n™ u 1p im ly, FuPtan?0-+ 5 Mw? sec? 0 zu i) - 222 x 100 ze i - (as? fysee? 0-1) x 100 and putting the values of @ and — , we get % of change in kinetic energy= - 40 % Wa * (a) Let the particles m, and m, move with velocities ¥y and V3 respectively. On the basis of solution of problem 1.147 (b) B= vy oH [7-7 1.175 1.176 91 As yl m, So, Be uViee where p= 2 mm, +m, (®) Again from 1.147 (b) zi Lye me? T= zuam gH [it~ H | zd So, T= SnOj+) From conservation of momentum Rm +P meme? , , ' so (F-P) = Pi- 2p, py’ 0050, + 7,'? = pr? From conservation of energy 212 2 Eliminating p,' we get ro, ™ ‘ of, _ m2 O=p (+ im, | 7 73 Pr0086; + Pi [1-7 This quadratic equation for p,’ has a real solution in terms of p, and cos 0, only if 2 mi ttn =| m mi 2 2 or sin? 0, <7 m m or sin®,<+ 22 or sinO2-—2 m m, This clearly implies (since only + sign makes sense) that sin 0, mag 2 m From the symmetry of the problem, the velocity of the disc A will be directed either in the initial direction or opposite to it just after the impact. Let the velocity of the disc A after the collision be ¥” and be directed towards right after the collision. It is also clear from the symmetry of problem that the discs B and C have equal speed (say v”) in the directions, shown. From the condition of the problem, 4D 50, sinO= Vio /2 (1) 2 For the three discs, system, from the conservation of linear momentum in the symmetry direction (towards right) my= 2mv"sinO+ mv! or, v= 2v"sinO+v' 2) 92 1.177 1.178 From the definition of the coefficeint of restitution, we have for the discs A and B (or C) v"—v' sin ® em “ysin 0-0 But e= 1, for perfectly elastic collision, So, vsin 0 = v—v'sin@ @) From (2) and (3), A in? yn Y= 2 sin? 0) ” (1 +2sin? 6) var -2) . oon {using (1)} Hence we have, y(n? = ve Vn =?) 6-1 Therefore, the disc A will recoil if n < VZ and stop if y= v2. Note : One can write the equations of momentum conservation along the direction per- pendicular to the initial direction of disc A and the consevation of kinetic energy instead of the equation of restitution. (a) Let a molecule comes with velocity V7 to strike another stationary molecule and just after collision their velocities become ¥”, and ¥”5 respectively. As the mass of the each molecule is same, conservation of linear momentum and conservation of kinetic energy for the system (both molecules) respectively gives : We P47 and vie vita From the property of vector addition it is obvious from the obtained Eqs. that Le, or Fy, = 0 (b) Due to the loss of kinetic energy in inelastic collision vi > v'? + v3 so, ¥°,-¥")>0 and therefore angle of divergence < 90°. Suppose that at time 4 the rocket has the mass m and the velocity 0”, relative to the reference frame, employed. Now consider the inertial frame moving with the velocity that the rocket has at the given moment. In this reference frame, the momentum increament that the rocket & ejected gas system acquires during time dt is, dp= mdvV+udtu= F dt aw Plo or, men Fw mite F- pi” 1.179 1.180 1.181 93 According to the question, F= Oand pu = — dm/dt so the equation for this system becomes, dv" dm ™ ae” ar" As dvi so, mdv= -udm. Integrating within the limits : mo Thus, v= uln—2 m As di") a so in vector form 7% -ia7In 2 ™ According to the question, F (external force) = 0 So, mae, ane ” dt” dt As a Li so, in scalar form, mdv= -udm wat dm or, “e-2 u ™ Integrating within the limits for m (1) dm v m om or, Ye -n 4S m u my ™ Hence, m= my en om) As F= 0, from the equation of dynamics of a body with variable mass; me PO yy are ee a) dt Now dv} {i"and since ii", ¥ we must have | dv"| = vod a (because vp is constant) where da. is the angle by which the spaceship turns in time dt. dm u dm So, -u——=vyda or, da= -——~ m Vo m ™ u fdm_ iu, (mo or, an -4 | SL 4/7 %yJ mv \m m 94 1.182 1.183 1.184 We have Sm —y os, d= - nat ™ ' Integrating fa- wu fa of, m= my- pt ™ 0 As ii’= 0 so, from the equation of variable mass system : (m-w) 222 Foo, Wo he FH ) I o, we (img - ws) > - a « fone f oan 0 Hence oe Fyn (— Bo" | my - Be Let the car be moving in a reference frame to which the hopper is fixed and at any instant of time, let its mass be m and velocity 7 Then from the general equation, for variable mass system. = =r odm m Fei o We write the equation, for our system as, dv > dm oe me FT as, Te 7 Q) So 4 iy -F and ve on integration. m But m= my + wt > Fr * laa Ht) mo(t+e] 7 F Thus the sought acceleration, w= 2. —__, my (: + m) my Let the length of the chain inside the smooth horizontal tube at an arbitrary instant is x. From the equation, rite Fee 1.185 95 as i= 0, Ftt w for the chain inside the tube haw= T where k= F q) Similarly for the overhanging part, AW r = 0 = Thus mw= F T or hhw= Mhg-T Q h From (1) and (2), Ang AG+h)w= hhg on, (+h 2~ hg B = or, (+h)v ta" gh, [As the fength ofthe chin ise the tube decreases with time, ds-= - de] or, vdve heh Integrating, [> dv= anf On (=k) 2 on Se ghin (i) or v= Y2eh In (i) Force moment relative to point O ; = dM oo Neo abt . Let the angle between M and N, a= 45° att = foe then 2 MN @+bG)-(2 dt) VE" [MINT Va +615 2b! wie bie Vas bie 2 Varig So, 2B = a? +b?ty oF, t= Vv £ (as tp cannot be negative) It is also obvious from the figure that the angle a is equal to 45° at the moment fo, when a= bi2,ie. fy = Va7B and N= 25 a 1.187 1.188 - J mvg gt? cos a (- i): mvy 8 t? cos 2 ‘Thus angular momentum at maximum height « vsina jeat reps 3 m(5)- ($e) sa? cose 37kg - m/s Alternate : Thus M (t) = ' ' M(@)= 0 50, Mo~ f Naw [ (Femz) ° 0 . -f [ (s+ 5a") 0 On the angular momentum of the disc relative to any of these points does not change in the given I process. (b) During the course of collision with wall the position of disc is same and is equal to 7, Obviously the increment in linear momentum of the ball Ap™ “2mv 08 an Here, AM = F7, x Ap’ 2mv cos a. and directed normally emerging from the plane of figure Thus |AM|= 2mvlcosa (a) The ball is under the influence of forces T and m g’at all the moments of time, while oe moving along a horizontal circle. Obviously the vertical component of T balance m g’and 1.189 1.190 97 so the net moment of these two about any point becoems zero. The horizontal component of 7) which provides the centripetal acceleration to ball is already directed toward the centre (C) of the horizontal circle, thus its moment about the point C equals zero at all the moments of time. Hence the net moment of the force acting on the ball about point C equals zero and that’s why the angular mommetum of the ball is conserved about the horizontal circle. (b) Let «be the angle which the thread forms with the vertical. Now from equation of particle dynamics : Tcosa= mg and Tsina = mo’ Isin a Hence on solving cos a= a a) o ‘As |M | is constant in magnitude so from figure. |AM|= 2M cos a where M~ |M,|= |M;| - [jo xmi"| = mv (as 717) Thus| AM |= 2 mv1cos a= 2. mw 1? sino. cos a _ 2mgl B_Y (uci ame (& (using 1). During the free fall time t= t = Ve , the reference point O moves in hoizontal direction (say towards right) by the distance ve In the translating frame as M (0) = 0, so AM=M,-7" : IW = (-Vuithj )xm[gtj-Vi] 23 = -mVg Cit mh (+k) J (9) - -mve(F)Femva cok) = -mVhE Hence [AM|= mVh The Coriolis force is.(2mv” x @). Here @ is along the z-axis (vertical). The moving disc is moving with velocity vy which is constant. The motion is along the x-axis say. Then the Coriolis force is along y-axis and has the magnitude 2m vq w. At time t, the distance of the centre of moving disc from is vot (along x-axis). Thus the torque N due to the coriolis force is N= 2mvoorvet along the z-axis. 1.191 L192 Hence equating tis 1 dM TT UmMat or M= myst? + constant. The constant is irrelevant and may be put equal to zero if the disc is originally set in motion from the point O. This discussion is approximate. The Coriolis force will cause the disc to swerve from straight line motion and thus cause deviation from the above formula which will be substantial for large t. If = radial velocity of the particle then the total energy of the particle at any instant is 1.2, Mm qmit sath nk a where the second term is the kinetic energy of angular motion about the centre O. Then the extreme values of r are determined by * = 0 and solving the resulting quadratic equation Hp EP +g we get From this we see that E=Kr+7) Q) where r, is the minimum distance from O and r, is the maximum distance. Then Joh + 2h} = kh) He 2k? fence, m= “2 ‘Note : Eq. (1) can be derived from the standard expression for kinetic energy and angular momentum in plane poler coordinates : 1 a1 aaa Ta5mi?+5mro M = angular momentum = mr’ The swinging sphere experiences two forces : The gravitational force and the tension of the thread. Now, it is. clear from the condition, given in the problem, that the moment of these forces about the vertical axis, passing through the point of suspension N,= 0. Con- sequently, the angular momentum M, of the sphere relative to the given axis (2) is constant. Thus my (Isin 0) = mv @) where m is the mass of the sphere and v is it s velocity in the position, when the thread x 2 forms an angle ~ with the vertical. Mechanical energy is also conserved, as the sphere is 1.193 1.194 1.195 99 under the influence if only one other force, i.e. tension, which does not perform any work, as it is always perpendicular to the velocity. So, A rmg+mg teos 0= 5m? @ From (1) and (2), we get, vy = Vigi7eos 0 Forces, acting on the mass m are shown in the figure. As N= mg? the net torque of these two forces about any fixed point must be equal to zero. Tension T, acting on the mass m is a central force, which is always directed towards the centre O. Hence the moment of force T is also zero about the point O and therefore the angular momentum of the particle m is conserved about O. Let, the angular velocity of the particle be w, when the separation between hole and particle m is r, then from the conservation of momentum about the point O, : m (7) ro = m(wr)r, or on 2 r Now, from the second law of motion for m, T= F=morr Hence the sought tension; On the given system the weight of the body m is the only force whose moment is effective about the axis of pulley. Let us take the sense of @ of the pulley at an arbitrary instant as the positive sense of axis of rotation (z-axis) As M,(0)= 0, 0, aM,= M,()= f Nae ' So, M,() J mg R dt= mg Rt ° Let the point of contact of sphere at initial moment (f= 0) be at O. At an arbitrary moment, the forces acting on the sphere are shown in the figure. We have normal reaction N, = mg sin @ and both pass through same line and the force of static friction passes through the point O, thus the moment about point 0 becomes zero. Hence mg sin a. is the only force which has effective torque about point O, and is given by |N |= mgRsina normally emerging from the plane of figure. As M(t 0)= 0, so, AM= Mo [Wa Hence, M(t) = Ne= mg R sin at 100 1.196 1.197 1.198 Let position vectors of the particles of the system be 77 and 7; with respect to the points O and O' respectively. Then we have, im HF +7 () where 75 is the radius vector of O' with respect to O. Now, the angular momentum of the system relative to the point O can be written as follows; Mo ¥ (xt)- D (7 «P+ (ex7) {using (1)] or, M- M+ Ge PY, where, pS) 7 Q) From (2), if the total linear momentum of the system, p’= 0, then its angular momen- tum does not depend on the choice of the point O. Note that in the C.M. frame, the system of particles, as a whole is at rest. On the basis of solution of problem 1.196, we have concluded that; “in the C.M. frame, the angular momentum of system of particles is independent of the choice of the point, relative to which it is determined” and in accordance with the problem, this is denoted by M. We denote the angular momentum of the system of particles, relative to the point O, by M, Since the internal and proper angular momentum Mf, in the CM. frame, does not depend on the choice of the point 0’, this point may be taken coincident with the point O of the K-frame, at a given moment of time. Then at that moment, the radius vectors of all the particles, in both reference frames, are equal (7; = 7;) and the velocities are related by the equation, Wee 4, @) where ¥7 is the velocity of C.M. frame, relative to the K-frame. Consequently, we may He Ym (eae Lima) mR) > miz, where m=). m;. or, Me f+ (rx mvt) = Ws (7x7) From conservation of linear momentum along the direction of incident ball for the system consists with colliding ball and phhere m yu ay wherev’ and v, are the velocities of ball and sphere 1 respectively after collision. (Remember that the collision is head on). As the collision is perfectly elastic, from the definition of co-efficeint of restitution, mvy= my’ + 1s (2) 1.199 101 Solving (1) and (2), we get, 40 directed towards 1i Vo 73 directed towa Is right, @>>--- } Mp In the C.M. frame of spheres 1 and 2 (Fig.) Se oe tae) pee) ow ce Py = ~P2 and |pyl=|P2] = wly-¥1 Also, Fro = -Fic, thus M = 2[7oxPr] c “ 4vy 0 AS rie L Bi, wo, Ba[} m2 | % (where 7 is the unit vector in the sense of 7c xP; ) mb ~ mv! Hence f=" In the CM. frame of the system (both the discs + spring), the linear momentum of the discs are related by the relation, p; = — D> at all the moments of time. where, Bim By= B= WM And the total kinetic energy of the system, T= hu v2, [See solution of 1.147 (b)] Bearing in mind that at the moment of maximum deformation of the spring, the projection of ¥,, along the length of the spring becomes zero, i.e. Vj) = 0. The conservation of mechanical energy of the considered system in the C.M. frame gives. 1 m 3) 3 nb) ra) q@ Now from the conservation of angular momentum of the system about the C.M., IB) Pe -1 oto x x or, mer” Teas” (+3) ~vof'-7) as x< where T= 12 years. m,= mass of ths Sun. Putting the values we get V, = 12-97 km/s 2 " vy 2xym, 2x rastninn = Z (280)".( 28) (34) em T = 24510 km/s? 1.202 1.203 1.204 1.205 1.206 103 Semi-major axis= (r + R)/2 r+R 2 It is sufficient to consider the motion be along a circle of semi-major axis for T does not depend on eccentricity. r+R\” 2nl 2 EO aV(r+R)/2ym, (again m, is the mass of the Sun) Hence T= We can think of the body as moving in a very elongated orbit of maximum distance R and minimum distance 0 so semi major axis = R/2. Hence if t is the time of fall then 2 3 2x) _(R2 22 72 (3)-(%2) « ere or t= T/4V2 = 365 / 4v2 = 645 days. T= 20R*?/Wym, If the distances are scaled down, R°” decreases by a factor 1°7and so docs m, . Hence T does not change. The double star can be replaced by a single star of mass moving about the centre of mass subjected to the force y m, m,/ 1°. Then T- 2a? _ 2a? / mM, M- YM Vim, m,/ ~~ ™, s - vm 0 72 = YM nm? or, r= (#5) (iy)? = WyM (1/2 xP 2x (@) The gravitational potential due to m, at the point of location of m, : => m, m, Van f Goa f Mae mahi) ¥ r ym my So, Uy = m,V.= -—— _ ym, m, Similarly Up= -17™ r 104 1.207 ym my Uy= Uy= U= -— (b) Choose the location of the point mass as the origin. Then the potential erfergy dU of an clement of mass au = Mae of the rod in the field of the point mass is M1 dU =~ ym de where x is the distance between the clement and the point. (Note that the rod and the point mass are on a straight line.) If then a is the distance of the nearer end of the rod from the point mass. ~------9— > x—>m l vee 14) The force of interaction is au Fe-3 mM 1 (ot yn “TTX 7 -3) “~Ga+) 1+ Minus sign means attraction. As the planet is under central force (gravitational interaction), its angular monientum is conserved about the Sun (which is situated at one of the focii of the ellipse) 4A So, mv,ry= mvyr, of, Vie @) A From the conservation of mechanical energy of the system (Sun + planet), umm 1 Ls tem tml ym, 14% (vm) 1 . or, a tage -[r, | 2 sine Ol Thus, vy= V2ym,7, 17 +r) Q) Hence M= mv,r.= mV2ym, 1, 79/ (r+) 1.208 1.209 105 From the previous problem, ifr, , r, are the maximum and minimum distances from the sun to the planet and v, , v2 are. the corresponding velocities, then, say, 12 Ym, Ex paw; - r ym, ry mm, ym, ymm, a Te eT Da Using Ea, (2) of 1.207] where 2a = major axis =r, +r, The same result can also be obtained directly by writing an equation analogous to Eq (1) of problem 1.191. E 1n 7+ = 2 mr or (Here M is angular momentum of the planet and m is its mass). For extreme position r= 0 and we get the quadratic we Er +ymmy - mn? The sum of the two roots of this equation are rytna- am Thus En eo constant 2a From the conservtion of angular momentum about the Sun. mvp MSinA= Mv, r= MV2r, Of, V4.7," V2r)= Vr Sina qa) From conservation of mechanical energy, ym,m_ 41 ym,m we 1 te =mvy- =5m zo 2 7 ym, vorsin?a 17 Using 1) Yo Ym - si ° 2 1 an n ne 2 on (8A) aearmn a’ 2 sina 0 So, 1 varasin’ 2 r[12V1-@-wnsiea | 1s ym, ™m, . 2% . @-w rm” Ym, where n= Yrq/ym,, (1m, is the mass of the Sun). 106 1.210 At the minimum separation with the Sun, the cosmic body’s velocity is perpendicular to 1.211 its position vector relative to the Sun. If r,;, be the sought minimum distance, from con- servation of angular momentum about the Sun (C). vol mvp l= mvraig OF, V= —— a Tooin From conservation of mechanical energy of tlie system (sun + cosmic body), 1 mm yy ymin - tym So, or, V8 rig + 2Y mM, Fain VEL? = 0 So , ~2y m2 Vay? m2 + avevg0 -ym,a Vy? m2 + ve PF , in ye 2ve wy Hence, taking positive root ig (ym, 3) [Vie g/m, F -1] ‘Suppose that the sphere has a radius equal to a. We may imagine that the sphere is made up of concentric thin spherical shells (layers) with radii ranging from 0 to a; and each spherical layer is made up of elementry bands (rings). Let us first calculate potential due to an elementry band of a spherjcal layer at the point of location of the point mass m (say point P) (Fig.). As all the points of the band are located at the distance / from the point P, so, a= - 12M (where mass of the band) ® aM=(sog7) (2eesing) (aa) - (2) sno ao Q And [?= a?+r?—2arcos® (3) Differentiating Eq. (3), we get Idl = ar sind do 4) Hence using above equations aon (Sar ja © 1.212 1.213 107 Now integrating this Eq. over the whole spherical layer ie aM dg=fag- aaa f ba So dg -14 © Equation (6) demonstrates that the potential produced by a thin uniform spherical layer outside the layer is such as if the whole mass of the layer were concentrated at it’s centre; Hence the potential due to the sphere at point P; o- fag= -Xfam=-™ 0 This expression is similar to that of Eq. (6) Hence thte sought potential energy of gravitational interaction of the particle m and the sphere, Ua mp= - wim (b) Using the Eq., G=- a G,= =" (wing Ea. 7) so G~ MF and Fa mG aT ® (The problem has already 4 clear hint in the answer Sheet of the problem book). Here we adopt a different method, Let m be the mass of the spherical layer, wich is imagined to be made up of rings. Ata point inside the spherical layer at distance r from the centre, the gravitational potential due to a ring element of radius a equals, d= - YT dl (Gee Eq. (5) of solution of 1.211) So, p= fdp= 2 fa--™ Q) Hence G= -2. oO. Hence gravitational field strength as well as field force becomes zero, inside a thin sphereical layer. One can imagine that the uniform hemisphere is made up of thin hemispherical layers of radii ranging from 0 to R. Let us consider such a layer (Fig.). Potential at point O, due to this layer is, 108 1.214 2 aq= —1. 3@ yap, where dm= —! | #27) r R (2/3)nR3\ 2 (This is because all points of each hemispherical shell are equidistant from O.) R 3yM 3yM Hence, = fdp= - 7 Srar Rh M Hence, the work done by the gravitational field N force on the particle of mass m, to remove it to infinity is given by the formula m A= mg, since = 0 at infinity. \] foes A AES (The work done by the external agent is - A.) In the solution of problem 1.211, we have obtained p and G due to a uniform shpere, at a distance r from it’s centre outside it. We have from Eqs. (7) and (8) of 1.211, M > M> o~ 14 ana G- Ur a) Accordance with the Eq. (1) of the solution of 1.212, potential due to a spherical shell of radius a, at any point, inside it becomes p= 4. const. and G,= -22- 0 ®) a or For a point (say P) which lies inside the uniform solid sphere, the potential @ at that point may be represented as a sum. Ponte P1*P2 where @, is the potential of a solid sphere having radius r and @, is the potential of the layer of radii r and R. In accordance with equation (A) y¥(_M__4, 3) _yM 2 MAF 33 Re” r((4/3)xR R The potential @, produced by the layer (thick shell) is the same at all points inside it. The potential @, is easiest to calculate, for the point positioned at the layer’s centre. Using Eq. (B) where dM= —! 4 ny? ar= (2M) Pa 73) xR R is the mass of a thin layer beeen the radii 7 and r+ dr. Thus page 1+ 82" (sa) >-5) © 1.215 109 From the Eq. G,~ =32 yMr oR or G- -M -v$np7F R? (where p= 3 M __ i the density of the sphere) ©) =xR? 3 The plots @ (r) and G (r) fora uniform sphere of radius R are shown in figure of answersheet. Alternate : Like Gauss’s theorem of electrostatics, one can derive Gauss’s theorem for ANY Mrciosea » For calculation of G at a point gravitation in the form ¢ G-dS= inside the sphere at a distance r from its centre, let us consider a Gaussian surface of radius r, Then, G4nr= -4mrfal on G,= “ir Hence, = -IMie _y4anpr 5 R (473) xR . R, . So, o- f care f rare | May r 2 - 43-2 ° 2R ( a) And from Gauss’s theorem for outside it : G,4nr*= -4nyM ot G,= -1 r Thus of Gare -m , Treating the cavity as negative mass of density ~ p in a uniform sphere density +p and using the superposition principle, the sought field strength is : d-G +d, a. 40 oe 4 = or Gu -Znypr, + -FyR(-p) 7 (where 77 and 7” are the position vectors of an orbitrary point P inside the cavity with respect to centre of sphere and cavity respectively.) q-7 Thus = -Fav0 (7 )= Saver 110 1.216 1.217 We partition the solid sphere into thin spherical layers and consider a layer of thickness dr lying at a distance r from the centre of the ball. Each spherical layer presses on the layers within it. The considered layer is attracted to the part of the sphere lying within it (the outer part does not act on the layer). Hence for the considered layer dp4nr*= dF ‘43 v3" »)anr2arp) or, aP4nr?= ~~, ___ r (where p is the mean density of sphere) o, dp= Snyptrdr 2x R Thus p= f ap- yp? (R?~r?) (The pressure.must vanish at r = R.) o, p= z(0 ~@?/R) yM7/ a4, Putting p= M/(4/3) xR? Putting r= 0, we have the pressure at sphere’s centre, and treating it as the Earth where mean density is equal to p= 5:5 x 10°kg/m> and R= 64x 107 km we have, p= 1-73x10" Pa or 1-72 x 10° atms. (@) Since the potential at each point of a spherical surface (shell) is constant and is equal top=- um [as we have in Eq. (1) of solution of problem 1.212] We obtain in accordance with the equation i 1 U- Sfang~ 50 f dm Lym). um 2\" R]"" ~2R (The factor } is needed otherwise contribution of different mass elements is counted twice.) (b) In this case the potential inside the sphere depends only on r (see Eq. (C) of the solution of problem 1.214) 2 - 31m, ° 2R |* ra Here dm is the mass of an elementry spherical layer confined between the radii rand r+dr: dm= (4nr?drp)= (ae )# lM > wlf (3m) 2 3ym r? S( jre{- 2R (-s)] 0 After integrating, we get _3ye u- SR 1218 Leto= ‘V "SE « circular frequency of the satellite in the outer orbit, y= VCE, = circular frequency of the satellite in the inner orbit. (r-Ary So, relative angular velocity = cy # « where ~ sign is to be taken when the satellites are moving in the same sense and + sign if they are moving in opposite sense. Hence, time between closest approaches 2m 0 2m { 45 days (5 = 0) “ato” Vi, [BE 0-80 hour (6 = 2) where 6 is 0 in the first case and 2 in the second case. YM _ 667x107" x 5-96 x 10% o,- 1219 1" (RT (637% 10)" = 98 m/s? 2 2 ap. (2% 2x22 . 6 2 one or- (37) x (sexsi wi) 63710 0-034 m/s M ese and a, TMS. 667x410 X197X10" 5910-9 m/s? Roem (149-50 x 108 x 10°)? Then @, ¢ 2: 3 = 1: 00034 : 0-0006 1.220 Let A be the sought height in the first case. so 8 1M 100 R+hy yM__g 112 1.221 1.222 1.223 -2 or Poh 100" (1*R From the statement of the problem, it is obvious that in this case h< Tx Frm rx Fey I= N/F= (where F.= > F,= resultant force) or, Nya = 7x F., Nye ‘Thus length of the arm, f= = Fre Here obviously [F,, |= 2F and it is directed toward right along AC. Take the origin at C. Then about C, viaF+ wre V2 aF ) directed normally into the plane of figure. (Here a = side of the square.) Thus N= F a directed into the plane of the figure. _ FUND) | = 4 nas? Hence OF som 7 5 sin 4s Thus the point of application of force is at the ‘nd point of the side BC. 1238 1.239 119 (a) Consider a strip of length dr at a perpendicular distance x from the axis about which we have to find the moment of inertia of the rod. The elemental mass of the rod equals dm = Tae Moment of inertia of this element about the axis dl= dmx?= 7 dex? ‘Thus, moment of inertia of the rod, as a whole about the given axis : 2 mio ml I J Tt em 5 (b) Let us imagine the plane of plate as xy plane taking the origin at the intersection point of the sides of the plate (Fig.). a Obviously I= f dmy? dx * nm 2 -f (32) oO - ma? 3 2 Similarly I= me Hence from perpendicular axis theorem mi 2.42 Leakth=3 (4 +b ) which is the sought ,moment of inertia. (@) Consider an elementry disc of thickness dr. Moment of inertia of this element about the z-axis, passing through its C.M. 2 2 at, = (am) = psax & cc Qa where p = density of the material of the plate and S = area of cross section of ihe plate. Thus the sought moment of inertia 4 ’ SR? R? Le PSR f axe = pSb oad 2 - FpbR*(ass= =R?)

You might also like